Day-556 | Daily MCQs | UPSC Prelims | ECONOMICS

Day-556

Time limit: 0

Quiz-summary

0 of 5 questions completed

Questions:

  1. 1
  2. 2
  3. 3
  4. 4
  5. 5

Information

DAILY MCQ

You have already completed the quiz before. Hence you can not start it again.

Quiz is loading...

You must sign in or sign up to start the quiz.

You have to finish following quiz, to start this quiz:

Results

0 of 5 questions answered correctly

Your time:

Time has elapsed

You have reached 0 of 0 points, (0)

Categories

  1. Not categorized 0%
  1. 1
  2. 2
  3. 3
  4. 4
  5. 5
  1. Answered
  2. Review
  1. Question 1 of 5
    1. Question

    1. In the context of economics, which of the following statements correctly describes the ‘Lewis Model’?

    Correct

    Answer: A
    Explanation:
    • In 1954, economist William Arthur Lewis put forth the “Economic Development with Unlimited Supplies of Labor”. For this work Lewis won the Nobel Prize in Economics in 1979.
    • The crux of the model suggested that surplus labor in agriculture could be redirected to the manufacturing sector by offering wages just high enough to attract workers away from the farm.
    • This shift, in theory, would stimulate industrial growth, enhance productivity, and lead to economic development.
    Challenges in Implementation of Lewis Model in India:
    • Obstacles of Low Wages: Low wages and inadequate social security in urban manufacturing facilities fails to entice rural agricultural laborers to relocate, given the high costs of urban living, posing a hurdle to the implementation of the Lewis model.
    • Technological Shift in Manufacturing: Manufacturing industries are increasingly capital-intensive, relying on labor-displacing technologies like robotics and artificial intelligence.
    • This transition restricts the absorptive capacity of labor-intensive sectors to accommodate surplus agricultural workers.
    • Disguised Unemployment: India faces a scenario of disguised unemployment in the agricultural sector, where a surplus of workers is engaged in activities that do not significantly contribute to increased productivity or income.
    • This surplus labor situation complicates the transition of workers to other sectors.
    • Skill Mismatch: There exists a mismatch between the skills demanded by the industries and the skills possessed by the workforce.
    • The education system might not adequately prepare individuals for the demands of the modern job market, resulting in a skill gap that impedes labor absorption in industries.
    • Overemphasis on White-Collar Jobs: Societal perceptions often prioritize white-collar jobs over vocational or technical skills.
    • This bias against blue-collar work can limit the workforce available for skilled trade positions and technical jobs, affecting industrial growth.

    Incorrect

    Answer: A
    Explanation:
    • In 1954, economist William Arthur Lewis put forth the “Economic Development with Unlimited Supplies of Labor”. For this work Lewis won the Nobel Prize in Economics in 1979.
    • The crux of the model suggested that surplus labor in agriculture could be redirected to the manufacturing sector by offering wages just high enough to attract workers away from the farm.
    • This shift, in theory, would stimulate industrial growth, enhance productivity, and lead to economic development.
    Challenges in Implementation of Lewis Model in India:
    • Obstacles of Low Wages: Low wages and inadequate social security in urban manufacturing facilities fails to entice rural agricultural laborers to relocate, given the high costs of urban living, posing a hurdle to the implementation of the Lewis model.
    • Technological Shift in Manufacturing: Manufacturing industries are increasingly capital-intensive, relying on labor-displacing technologies like robotics and artificial intelligence.
    • This transition restricts the absorptive capacity of labor-intensive sectors to accommodate surplus agricultural workers.
    • Disguised Unemployment: India faces a scenario of disguised unemployment in the agricultural sector, where a surplus of workers is engaged in activities that do not significantly contribute to increased productivity or income.
    • This surplus labor situation complicates the transition of workers to other sectors.
    • Skill Mismatch: There exists a mismatch between the skills demanded by the industries and the skills possessed by the workforce.
    • The education system might not adequately prepare individuals for the demands of the modern job market, resulting in a skill gap that impedes labor absorption in industries.
    • Overemphasis on White-Collar Jobs: Societal perceptions often prioritize white-collar jobs over vocational or technical skills.
    • This bias against blue-collar work can limit the workforce available for skilled trade positions and technical jobs, affecting industrial growth.

  2. Question 2 of 5
    2. Question

    2. Consider the following statements:
    1. Mutual funds invest money in equities only.
    2. Mutual funds can reduce the volatility in return on investment.
    3. Infrastructure Investment Trust (InvIT) and Real estate Investment Trust (ReITs) are similar to mutual funds.
    How many of the statements given above are correct?

    Correct

    Answer: B
    Explanation:
    Statement 1 is incorrect: Mutual funds collect money from investors and invest the money, on their behalf, in different securities (debt, equity or both).
    Statement 2 is correct: Equity mutual funds pool money from multiple investors to create a diversified portfolio of stocks. Due to risk diversification, the volatility in returns generated from mutual funds investments are generally lower than that of direct equity investing.
    Statement 3 is correct: Infrastructure investment Trust (InvIt) and Real estate Investment Trust(ReITs) are similar to mutual funds. This is because both REITs and InvITs pool money from a large number of investors and invest money on physical assets to generate income which is distributed as dividend.
    Additional information:
    ● An equity mutual fund is a type of mutual fund that primarily invests in stocks or equities. It allows individual investors to pool their money together and invest in a diversified portfolio of stocks selected and managed by professional fund managers.
    ● The objective of equity mutual funds is to provide investors with the opportunity to participate in the potential returns and dividends generated by a wide range of publicly traded companies.
    ● Equity funds are actively managed by experienced fund managers who conduct research and analysis to make investment decisions. These professionals aim to select stocks that they believe will generate positive returns for investors.

    Incorrect

    Answer: B
    Explanation:
    Statement 1 is incorrect: Mutual funds collect money from investors and invest the money, on their behalf, in different securities (debt, equity or both).
    Statement 2 is correct: Equity mutual funds pool money from multiple investors to create a diversified portfolio of stocks. Due to risk diversification, the volatility in returns generated from mutual funds investments are generally lower than that of direct equity investing.
    Statement 3 is correct: Infrastructure investment Trust (InvIt) and Real estate Investment Trust(ReITs) are similar to mutual funds. This is because both REITs and InvITs pool money from a large number of investors and invest money on physical assets to generate income which is distributed as dividend.
    Additional information:
    ● An equity mutual fund is a type of mutual fund that primarily invests in stocks or equities. It allows individual investors to pool their money together and invest in a diversified portfolio of stocks selected and managed by professional fund managers.
    ● The objective of equity mutual funds is to provide investors with the opportunity to participate in the potential returns and dividends generated by a wide range of publicly traded companies.
    ● Equity funds are actively managed by experienced fund managers who conduct research and analysis to make investment decisions. These professionals aim to select stocks that they believe will generate positive returns for investors.

  3. Question 3 of 5
    3. Question

    3. With reference to various curves in economics, consider the following pairs:
    Economics curves – Relationships between variables
    1. Laffer Curve – Inflation rate and Unemployment rate
    2. Phillips Curve – Tax revenue and Tax rate
    3. Lorenz Curve – National Income and Population Distribution
    How many of the above pairs are correctly matched?

    Correct

    Answer: A
    Explanation
    • Pair 1 is incorrect: Laffer curve explains the relationship between tax revenue and tax rate. It states that at lower as well as higher rate of tax, the tax revenue is low but tax revenue is high at the optimal rate of tax. According to the Laffer curve, if tax rates are increased above a certain level, then tax revenues can actually fall because higher tax rates discourage people from working, also there is high tax evasion.
    • Pair 2 is incorrect: The Phillips curve is an economic theory that inflation and unemployment have a stable and inverse relationship. Developed by William Phillips, it claims that with economic growth comes inflation, which in turn should lead to more jobs and less unemployment.
    • Pair 3 is correct: Lorenz Curve is a graphical distribution of wealth. It shows the proportion of income earned by any given percentage of the population.

    Incorrect

    Answer: A
    Explanation
    • Pair 1 is incorrect: Laffer curve explains the relationship between tax revenue and tax rate. It states that at lower as well as higher rate of tax, the tax revenue is low but tax revenue is high at the optimal rate of tax. According to the Laffer curve, if tax rates are increased above a certain level, then tax revenues can actually fall because higher tax rates discourage people from working, also there is high tax evasion.
    • Pair 2 is incorrect: The Phillips curve is an economic theory that inflation and unemployment have a stable and inverse relationship. Developed by William Phillips, it claims that with economic growth comes inflation, which in turn should lead to more jobs and less unemployment.
    • Pair 3 is correct: Lorenz Curve is a graphical distribution of wealth. It shows the proportion of income earned by any given percentage of the population.

  4. Question 4 of 5
    4. Question

    4. Consider the following statements regarding Gross Domestic Product (GDP)and Gross Value Added (GVA):
    1. GDP gives the picture of an economy from the demand perspective while GVA reflects the state of economic activity from the supply side.
    2. GDP includes the value of intermediate goods whereas GVA does not.
    Which of the statements given above is/are correct?

    Correct

    Answer: A
    Explanation:
    • Statement 1 is correct: GVA is the value added to the product to enhance the various aspects of the product whereas GDP is the total amount of products produced in the country. While GDP gives the picture from the consumers’ side or demand perspective, GVA gives a picture of the state of economic activity from the producers’ side or supply side. Both measures need not match because of the difference in treatment of net taxes.
    • Statement 2 is incorrect: Both GDP as well as GVA do not include the value of intermediate goods. GDP is calculated by adding Net Taxes (taxes earned by the government – subsidies paid by the government) to the GVA. GDP fails to gauge the real economic scenario because a sharp increase in the output can be due to higher tax collections which could be on account of better compliance or coverage, rather than the real output situation. As a result, GVA is a more precise measure of the value added by each sector of the economy.
    Additional information:
    ● GDP is the sum of private consumption, gross investment in the economy, government investment, government spending and net foreign trade (the difference between exports and imports).
    ● GDP = private consumption + gross investment + government investment + government spending + (exports-imports)
    ● GDP measures the value of a country’s final goods and services (those purchased by the final user) generated in a specific time period (say, a quarter or a year). It includes all of the output produced within a country’s borders.
    GDP = Private consumption + gross investment + government investment + government spending + (exports – imports)
    ● Private Consumption Expenditure refers to the value of all goods and services purchased for consumption by households.
    ● Government Consumption Expenditure refers to the value of all goods and services purchased for consumption by the government.
    ● Gross Investment refers to the total value of all capital investments made in the economy.
    The GVA of a sector is defined as the value of output minus the value of its intermediary inputs. This “value added” is distributed among the primary production factors, labour and capital. By examining GVA growth, one may determine which sectors of the economy are doing well and which are struggling.
    Calculation of GVA
    ● GVA is the sum of a country’s GDP and net of subsidies and taxes in the economy at the macro level, according to national accounting.
    ● Gross Value Added = GDP + product subsidies – product taxes
    ● Previously, India measured GVA at ‘factor cost’ until a new methodology was implemented, in which GVA at ‘basic prices’ became the primary measure of economic output.
    ● GVA at basic prices will include production taxes and exclude production subsidies.
    ● GVA at factor cost included no taxes and excluded no subsidies

    Incorrect

    Answer: A
    Explanation:
    • Statement 1 is correct: GVA is the value added to the product to enhance the various aspects of the product whereas GDP is the total amount of products produced in the country. While GDP gives the picture from the consumers’ side or demand perspective, GVA gives a picture of the state of economic activity from the producers’ side or supply side. Both measures need not match because of the difference in treatment of net taxes.
    • Statement 2 is incorrect: Both GDP as well as GVA do not include the value of intermediate goods. GDP is calculated by adding Net Taxes (taxes earned by the government – subsidies paid by the government) to the GVA. GDP fails to gauge the real economic scenario because a sharp increase in the output can be due to higher tax collections which could be on account of better compliance or coverage, rather than the real output situation. As a result, GVA is a more precise measure of the value added by each sector of the economy.
    Additional information:
    ● GDP is the sum of private consumption, gross investment in the economy, government investment, government spending and net foreign trade (the difference between exports and imports).
    ● GDP = private consumption + gross investment + government investment + government spending + (exports-imports)
    ● GDP measures the value of a country’s final goods and services (those purchased by the final user) generated in a specific time period (say, a quarter or a year). It includes all of the output produced within a country’s borders.
    GDP = Private consumption + gross investment + government investment + government spending + (exports – imports)
    ● Private Consumption Expenditure refers to the value of all goods and services purchased for consumption by households.
    ● Government Consumption Expenditure refers to the value of all goods and services purchased for consumption by the government.
    ● Gross Investment refers to the total value of all capital investments made in the economy.
    The GVA of a sector is defined as the value of output minus the value of its intermediary inputs. This “value added” is distributed among the primary production factors, labour and capital. By examining GVA growth, one may determine which sectors of the economy are doing well and which are struggling.
    Calculation of GVA
    ● GVA is the sum of a country’s GDP and net of subsidies and taxes in the economy at the macro level, according to national accounting.
    ● Gross Value Added = GDP + product subsidies – product taxes
    ● Previously, India measured GVA at ‘factor cost’ until a new methodology was implemented, in which GVA at ‘basic prices’ became the primary measure of economic output.
    ● GVA at basic prices will include production taxes and exclude production subsidies.
    ● GVA at factor cost included no taxes and excluded no subsidies

  5. Question 5 of 5
    5. Question

    5. Consider the following statements regarding Nominal Effective Exchange Rate (NEER) and Real Effective Exchange Rate (REER):
    1. A Nominal Effective Exchange Rate (NEER) adjusted for inflation equals its Real Effective Exchange Rate (REER).
    2. An increase in a nation’s REER is an indication that its exports are becoming more expensive and its imports are becoming cheaper.
    Which of the statements given above is/are correct?

    Correct

    Answer: C
    Explanation:
    Statement 1 is correct: A Nominal Effective Exchange Rate (NEER) adjusted for inflation equals its Real Effective Exchange Rate (REER). The important aspect of NEER is that it is not adjusted for inflation. Thus, it does not say anything about the real strength of a currency; it is just a relative value. REER is considered a more accurate measure to gauge a currency’s strength. The Real Effective Exchange Rate (REER) is the NEER adjusted by inflation differentials between the domestic currency and that of the trading partners. Thus, it says about the real strength of a domestic currency against a basket of currencies, unlike NEER.
    Statement 2 is correct: An increase in a nation’s REER is an indication that its exports are becoming more expensive and its imports are becoming cheaper.
    Additional information:
    ● The Real Effective Exchange Rate (REER) is the weighted average of a country’s currency in relation to an index or basket of other major currencies.
    ● The weights are determined by comparing the relative trade balance of a country’s currency against that of each country in the index.
    ● The Nominal Effective Exchange Rate (NEER) is an index of the weighted average of bilateral exchange rates of home currency with respect to a basket of currencies of trading partners. It is also known as the trade-weighted currency index.
    ● An increase in NEER indicates that the domestic currency has appreciated against the basket of currencies whereas a decrease indicates a relative depreciation.
    NEER and REER in the Indian context
    ● The Reserve Bank compiles and disseminates indices of NEER and REER of the Indian rupee.
    ● The latest series was constructed with the base year 2015-16. The new indices of NEER and REER replaced the old series from 2004-05 onwards.
    ● The coverage of the NEER and REER basket has been expanded from 36 to 40 currencies. This was in view of the growing importance of emerging markets and developing economies in India’s foreign trade and to better reflect shifts in external competitiveness.
    ● The selection of currencies for NEER and REER in the new series is based on two criteria:
    ● Trading partners with extremely high and volatile inflation are excluded. This is because their currencies tend to experience rapid nominal declines, undermining the stability of the NEER and REER indices. This obscures their usefulness in the assessment of external competitiveness.
    ● Data on inflation and exchange rates of trading partners should be available on a regular basis.

    Incorrect

    Answer: C
    Explanation:
    Statement 1 is correct: A Nominal Effective Exchange Rate (NEER) adjusted for inflation equals its Real Effective Exchange Rate (REER). The important aspect of NEER is that it is not adjusted for inflation. Thus, it does not say anything about the real strength of a currency; it is just a relative value. REER is considered a more accurate measure to gauge a currency’s strength. The Real Effective Exchange Rate (REER) is the NEER adjusted by inflation differentials between the domestic currency and that of the trading partners. Thus, it says about the real strength of a domestic currency against a basket of currencies, unlike NEER.
    Statement 2 is correct: An increase in a nation’s REER is an indication that its exports are becoming more expensive and its imports are becoming cheaper.
    Additional information:
    ● The Real Effective Exchange Rate (REER) is the weighted average of a country’s currency in relation to an index or basket of other major currencies.
    ● The weights are determined by comparing the relative trade balance of a country’s currency against that of each country in the index.
    ● The Nominal Effective Exchange Rate (NEER) is an index of the weighted average of bilateral exchange rates of home currency with respect to a basket of currencies of trading partners. It is also known as the trade-weighted currency index.
    ● An increase in NEER indicates that the domestic currency has appreciated against the basket of currencies whereas a decrease indicates a relative depreciation.
    NEER and REER in the Indian context
    ● The Reserve Bank compiles and disseminates indices of NEER and REER of the Indian rupee.
    ● The latest series was constructed with the base year 2015-16. The new indices of NEER and REER replaced the old series from 2004-05 onwards.
    ● The coverage of the NEER and REER basket has been expanded from 36 to 40 currencies. This was in view of the growing importance of emerging markets and developing economies in India’s foreign trade and to better reflect shifts in external competitiveness.
    ● The selection of currencies for NEER and REER in the new series is based on two criteria:
    ● Trading partners with extremely high and volatile inflation are excluded. This is because their currencies tend to experience rapid nominal declines, undermining the stability of the NEER and REER indices. This obscures their usefulness in the assessment of external competitiveness.
    ● Data on inflation and exchange rates of trading partners should be available on a regular basis.

window.wpAdvQuizInitList = window.wpAdvQuizInitList || []; window.wpAdvQuizInitList.push({ id: '#wpAdvQuiz_595', init: { quizId: 595, mode: 0, globalPoints: 10, timelimit: 0, resultsGrade: [0], bo: 0, qpp: 0, catPoints: [10], formPos: 0, lbn: "Finish quiz", json: {"2769":{"type":"single","id":2769,"catId":0,"points":2,"correct":[1,0,0,0]},"2770":{"type":"single","id":2770,"catId":0,"points":2,"correct":[0,1,0,0]},"2771":{"type":"single","id":2771,"catId":0,"points":2,"correct":[1,0,0,0]},"2772":{"type":"single","id":2772,"catId":0,"points":2,"correct":[1,0,0,0]},"2773":{"type":"single","id":2773,"catId":0,"points":2,"correct":[0,0,1,0]}} } });




LEGISLATIVE DECLINE: ON GOVERNMENT’S ACTIONS AND A DISREGARD FOR DELIBERATIVE DEMOCRACY

THE CONTEXT: The security breach in Parliament featuring a theatrical attempt by individuals to highlight an issue of public importance and the Union government’s response to it have been deeply problematic.

MORE ON THE NEWS:

  • The government has hindered any debate over this issue in Parliament which lead to an unprecedented high number of suspensions of Opposition legislators which disregard deliberative democracy.
  • Over 90 opposition MPs suspended so far for causing ruckus and for disrupting parliamentary proceedings.
  • Unlike in the two Lok Sabhas (2004-14), when even ruling party legislators, including rebels, were suspended for unruliness, this time only opposition members have been subject to suspension and also this is for far less severe offences since 2014.

ISSUES:

  • Denial of legislative debate: Denial of legislative debate further affects the democratic processes in the country. Legislative business and parliamentary work have been often given short time while one-upmanship through the use of suspensions, have dominated proceedings.
  • Underutilising parliamentary committee: Over the course of recent parliamentary sessions, there have been multiple attempts at oppressing the opposition. There have been attempts at getting Bills passed without adequate discussion and under-utilising standing and parliamentary committees.
  • Targeting dissenters: One of the issue arising is misuse of the draconian Unlawful Activities (Prevention) Act to indiscriminately target dissenters, such as the protesters who threw canisters and raised slogans in Parliament. It has also fallen into the recent pattern of a deliberate equation of dissent with terror.
  • International image: Such actions in parliament in India have compelled global democracy reports by research institutions such as V-Dem Institute to characterise India’s democracy as an “electoral autocracy”. Also, this has led the U.S.-based Freedom House, that measures civil and political liberties, to declare India as “partially free”.

THE WAY FORWARD

  • Code of conduct of MPs and MLAs: There must be strict adherence to the code of conduct for MPs and MLAs so that there is least disruption of proceedings in parliament.
  • Strong legislative oversight: A strong legislative oversight system ensures accountable government and the realization of the welfare state’s promise. For an effective parliamentary democracy, strict parliamentary supervision is a must.
  • Effective discussion: The hallmark of a functioning democracy is deliberation, where elected legislators debate and discuss issues of public importance. A thorough deliberation can be done by efficient utilisation of parliamentary and standing committees to delve into the proposed legislation.
  • Regulating suspension of parliamentarians: Parliament can amend its rules to give MPs more power when confronting the government and enable its committees to play a larger role in the legislative process. The Speaker’s adjudicating power over anti-defection legislation could be handed to the other agencies like Election Commission of India for impartial decision of suspensions.

THE CONCLUSION:

Parliament is regarded as the highest institutions in the country in which representative democracy is implemented. However, the recent actions by the government contribute to the backsliding of democracy in India, making it a matter of serious concern. There is a need for further strengthening the Parliament to prevent disruption of its proceedings and to deepen its role as the forum for deliberation on critical national issues.

PREVIOUS YEAR QUESTIONS

Q.1 Explain the structure of the Parliamentary Committee system. How far have the financial committees helped in the institutionalisation of the Indian Parliament? (2023)

Q.2 Discuss the role of Presiding Officers of state legislatures in maintaining order and impartiality in conducting legislative work and in facilitating best democratic practices. (2023)

MAINS PRACTICE QUESTION

Q.1 The functioning of Parliament has deteriorated over time, which is not a good sign for a healthy democracy. Comment.

Source: https://www.thehindu.com/opinion/editorial/legislative-decline-the-hindu-editorial-on-governments-actions-and-a-disregard-for-deliberative-democracy/article67651320.ece




GLOBAL COAL DEMAND DYNAMICS AND RENEWABLE ENERGY TRANSITION

TAG: GS 3: ECONOMY

THE CONTEXT: Despite reaching an all-time high in production this year, global coal demand is expected to decline by 2026 as per a report by the International Energy Agency (IEA).

CURRENT STATE OF GLOBAL COAL DEMAND

  • Production vs. Projected Decline in Demand
    • Despite reaching an all-time high in production, the International Energy Agency (IEA) projects a decline in global coal demand by 2026.
    • The report attributes this anticipated reduction to a shift towards renewable energy sources and a saturation of demand from China, while highlighting India’s pivotal role in sustaining coal demand until 2026.
  • Regional Disparities in Demand
    • The forecasted decline in coal demand conceals regional differences.
    • While the European Union and the United States are expected to witness a significant drop in demand by 20% each, India and China are projected to experience an increase in demand by 8% and 5%, respectively.
    • It is primarily due to electricity needs and reduced hydropower generation.

FACTORS INFLUENCING THE DECLINE IN COAL DEMAND

  • Renewable Energy Expansion
    • The anticipated decrease in coal demand is underpinned by the burgeoning capacity of renewable energy sources.
    • The report emphasizes the substantial deployment of low-cost solar photovoltaic systems and the projected increase in nuclear energy generation, especially in China, India, and the European Union.
  • Climate and Weather Conditions
    • The IEA report correlates the decline in coal demand with climate factors.
    • It anticipates the transition from El Nino to La Nina, potentially leading to improved rainfall in Asia during 2024-2026, thereby augmenting hydropower generation and reducing the reliance on coal.

COAL’S ENVIRONMENTAL IMPACT AND POLICY IMPLICATIONS

  • CO2 Emissions and Climate Targets
    • Coal remains a dominant energy source but is also the largest contributor to carbon dioxide (CO2) emissions.
    • The report highlights the necessity of reducing ‘unabated’ coal use to meet international climate targets, emphasizing the United Nations Framework Convention on Climate Change’s agreement to decrease coal emissions by nearly 95% between 2020-2050 to limit global temperature rise.
  • Shift in Climate Policy
    • The IEA underscores the significance of global climate policy in steering away from unabated coal usage.
    • Efforts to limit temperature increases to 1.5°C by the end of the century mandate a substantial reduction in coal emissions, signalling a pivotal moment for renewable energy expansion.

GLOBAL COAL PRODUCTION TRENDS

  • Production Records and Major Producers
    • China, India, and Indonesia, the world’s three largest coal producers, are expected to set production records in 2023, collectively contributing over 70% of global coal production.
    • Despite rising production levels, the report highlights the divergence between production surges and the projected decline in demand.

INTERNATIONAL ENERGY AGENCY

  • It is an autonomous inter-governmental organisation within the OECD framework.
  • It works with governments and industry to shape a secure and sustainable energy future for all.
  • It was founded in 1974 to ensure the security of oil supplies.
  • It was created in response to the 1973-1974 oil crisis when an oil embargo by major producers pushed prices to historic levels and exposed the vulnerability of industrialised countries to dependency on oil imports.
  • It consists of 31 member countries and eleven association countries.
  • A candidate country to the IEA must be a member country of the Organisation for Economic Co-operation and Development (OECD).
  • India joined this organization in 2017 as an Associate member.

CONCLUSION: THE TRANSITION AWAY FROM COAL

  • The IEA report points to a transformative period marked by a structural decline in coal demand, driven by sustained expansion in renewable energy technologies.
  • It emphasizes the pivotal role of Asia’s renewable energy expansion in determining the pace of coal’s phase-out.
  • However, meeting international climate targets necessitates accelerated efforts towards reducing coal emissions.

SOURCE: https://www.thehindu.com/news/international/global-coal-demand-expected-to-decline-by-2026-iea-report/article67647691.ece




THE EVOLUTION AND FUTURE OF CRISPR THERAPEUTICS

TAG: GS 3: SCIENCE AND TECHNOLOGY

THE CONTEXT: The recent approvals by regulatory agencies in the UK and the US for CRISPR-based therapies targeting sickle-cell disease and β-thalassemia mark a transformative era in medicine.

EXPLANATION:

  • These inherited blood disorders affect millions globally, and the approvals signify a shift towards addressing their molecular basis rather than just symptom management.

THE EVOLUTION OF CRISPR TECHNOLOGY

  • Origins of CRISPR
    • CRISPR, an acronym for Clustered Regularly Interspaced Short Palindromic Repeats, originated from the discovery of DNA elements in 1993 by Spanish researchers.
    • These elements were found in archaea and later in bacterial genomes.
    • Initially thought to be a part of the bacterial immune system against viruses, it was revealed that CRISPR, combined with CRISPR-associated proteins (Cas), functioned as an antiviral defense mechanism.
  • Milestone Discoveries
    • The groundbreaking work of Emmanuelle Charpentier, Jennifer Doudna, and Virginijus Siksnys led to key discoveries.
    • In 2010, it was demonstrated that CRISPR, specifically with Cas9 proteins, could cut DNA at precise points.
    • The identification of RNA molecules guiding Cas9 to specific genomic positions further revolutionized its potential.
    • This work culminated in the development of a programmable ‘molecular scissor’ capable of editing DNA accurately.
  • CRISPR-Cas9 Advancements
    • Subsequent research by Feng Zhang and George Church showcased CRISPR-Cas9’s ability to edit the genomes of eukaryotic organisms.
    • This innovation expanded its applications, ranging from genetic therapies to agricultural advancements.

CRISPR IN MEDICINE: CURRENT ACHIEVEMENTS AND FUTURE PROSPECTS

  • First-Generation Technologies
    • The approved CRISPR-based therapeutics represent first-generation technologies.
    • Though groundbreaking, they are continuously evolving to become more efficient and effective.
    • Novel approaches like base editing and prime editing hold immense promise for precise genome editing at the nucleotide level, addressing diseases like familial hypercholesterolemia.
  • Emerging Techniques and Challenges
    • Techniques like base editing and prime editing are showing potential, yet safety and accuracy issues persist.
    • Off-target events, where CRISPR systems inaccurately edit unintended parts of the genome, pose risks.
    • Balancing short-term benefits with long-term risks remains critical, especially as these therapies are still in early developmental stages.

THE FUTURE LANDSCAPE AND CONSIDERATIONS

  • Potential and Caution
    • While celebrating the transformative potential of CRISPR-based therapies like Casgevy, it’s essential to acknowledge potential risks.
    • Continued scrutiny and surveillance are imperative to identify and address unforeseen side effects.
    • The enormous promise of these technologies must be cautiously balanced with potential risks to ensure patient safety and ethical considerations.
  • Moving Forward
    • CRISPR technology has opened doors to a future where the correction of genetic anomalies is a reality.
    • The ongoing advancements in CRISPR-based therapies signify a promising era in medicine, holding immense potential to alleviate the suffering of millions affected by genetic diseases.

CONCLUSION

  • The approval of CRISPR-based therapies represents a significant leap in medical science, offering hope to patients with genetic disorders.
  • While these treatments mark a milestone, continuous research, vigilance, and technological advancements are crucial to maximize benefits while minimizing risks associated with genome editing technologies.

SOURCE: https://www.thehindu.com/sci-tech/science/crispr-casgevy-sickle-cell-disease-genetic-therapeutics-explained/article67641478.ece




FALI S NARIMAN WRITES: WHERE THE SUPREME COURT WENT WRONG ON ARTICLE 370

THE CONTEXT: The Supreme Court of India recently unanimously upheld the actions of the Indian government.  Though, abrogation of Article 370 can be held politically correct, concern arises as it lets the Centre get away with violating the constitution and federal principles.

MORE ON THE NEWS:

  • The decision has been upheld, in three judgments, one main and two concurring of the Constitution Bench of five judges of the Supreme Court which has facilitated a complete integration of Jammu and Kashmir into the Union of India.
  • The unanimous judgment would have been welcomed but this is not all that happened as is facing criticism because of below mentioned reasons.

ISSUES:

  • Against constitution and federal principles: Actions taken by the Centre was neither according to the provisions of the Constitution, nor in accordance with well-settled principles of federalism which is a basic feature of the Constitution.
  • Diminution without consent of state assembly: Under Article 370 of the Constitution, Article 3 was applied to the erstwhile state of Jammu and Kashmir. It came with a condition that its area would not be diminished by the executive nor by Parliament, without the consent of the J&K State Assembly. However, contrary to this assurance, a very substantial diminution of the area of the State of J&K held without the consent of the inhabitants of Jammu and Kashmir.
  • Change of status quo: Recent judgments led not only diminution of the area of the state of Jammu and Kashmir but its status too was unilaterally altered from state to Union Territory a situation not justified by any provision in the Constitution.
  • Threat to Federalism: The powers of the Union government with respect to states seem to be enhanced by the verdict, which could be politically unstable. As it could lead to threat to federal principles.
  • Concerns of Parliamentary Sovereignty: There are concerns that the  Union can impose the President’s Rule first and then substitute parliamentary approval in place of the Assembly even for highly divisive issues like the splitting of a state.
  • Powers of the President under Article 370(1) (d): The President’s power under the Article was not a “constituent power” but merely a power to apply provisions with “modifications and exceptions.”  Thus, abrogation of Article 370 without the consent of State’s Constituent Assembly is not right.
  • Violating constituent assembly: Another important safeguard for the state of Jammu & Kashmir had been set out in Article 370 (3) itself as enacted in1950. It states power of the President under Article 370 to declare the entire Article 370 inoperative can come into effect only if the precondition was fulfilled the recommendation of the Constituent Assembly of the state of Jammu & Kashmir. Disregarding this,
  • Wrong interpretation by the court: The conclusion of the Court that the recommendation of the Constituent Assembly is not binding on the President was based on the Court’s erroneous interpretation of Article 370 (3) as being in two separate parts. The court wrongly states that the power under Article 370 (3) did not cease to exist upon the dissolution of the Constituent Assembly of Jammu and Kashmir.

THE WAY FORWARD:

  • Revisit judgment: SC should act proactively and revisit its judgment of upholding the abrogation of Article 370 by the centre which was done without consulting state legislature.
  • Restoring normalcy: There is a need of restoring normalcy in the region for trust-building. It can be done by fostering dialogue and engaging local leaders for peaceful conduction of democratic elections and restoration of the statehood of J&K.
  • Ensuring Governance: There is a need for inclusive governance for addressing diverse aspirations of the region.
  • Economic development: The imminent focus of the government must be on promoting inclusive economic development of the region by effective implementation of the affirmative policies of the government.
  • Establishment of Truth and Reconciliation Commission: Justice Kaul, in his concurring opinion, suggested the establishment of a Truth and Reconciliation Commission in Jammu and Kashmir. This commission would be tasked with acknowledging and addressing alleged rights violations in the region, indicating a commitment to justice and reconciliation. It should be established as soon as possible.

THE CONCLUSION:

The present decision of the Supreme Court upholding the centre’s decision is not constitutionally correct. It undermines federalism and democratic processes in the region. There should be a collective action to ensure good governance in the region upholding the constitutional provisions.

PREVIOUS YEAR QUESTIONS

Q.1 To what extent is Article 370 of the Indian Constitution, bearing marginal note “Temporary provision with respect to the State of Jammu and Kashmir”, temporary? Discuss The future prospects of this provision in the context of Indian polity. (2016)

Q.2 The banning of ‘Jamaat-e-islaami’ in Jammu and Kashmir brought into focus the role of over- ground workers (OGWs) in assisting terrorist organizations. Examine the role played by OGWs inassisting terrorist organizations in insurgency affected areas. Discuss measures to neutralize the influence of OGWs. (2019)

MAINS PRACTICE QUESTION

Q.1 Critically examine the constitutional implications of the SC judgement upholding the centre’s decision of abrogation of Article 370 of the Constitution.

SOURCE: https://indianexpress.com/article/opinion/columns/supreme-court-verdict-on-article-370-fali-s-nariman-abbrogation-of-article-370-jammu-and-kashmir-9072109/

 




Day-555 | Daily MCQs | UPSC Prelims | SCIENCE AND TECHNOLOGY

Day-555

Time limit: 0

Quiz-summary

0 of 5 questions completed

Questions:

  1. 1
  2. 2
  3. 3
  4. 4
  5. 5

Information

DAILY MCQ

You have already completed the quiz before. Hence you can not start it again.

Quiz is loading...

You must sign in or sign up to start the quiz.

You have to finish following quiz, to start this quiz:

Results

0 of 5 questions answered correctly

Your time:

Time has elapsed

You have reached 0 of 0 points, (0)

Categories

  1. Not categorized 0%
  1. 1
  2. 2
  3. 3
  4. 4
  5. 5
  1. Answered
  2. Review
  1. Question 1 of 5
    1. Question

    1. With reference to the birth control pills, consider the following mechanisms:
    1. Stopping or reducing ovulation
    2. Thinning of the cervical mucus
    3. Thickening of uterus lining
    Birth control pills can target how many of the above mechanisms to prevent pregnancy in females?

    Correct

    Answer: A
    Explanation
    The birth control pill is a type of oral contraception that uses hormones to prevent pregnancy. The hormones in birth control pills prevent pregnancy by blocking conception when sperm fertilizes an egg. They also cause changes in the uterus so that it can’t support a pregnancy. Oral contraceptive pills are either combined estrogen-progesterone (also called combined oral contraceptive pill- COC) or progesterone-only pill (POP).
    ● Mechanism 1 is correct: One of the mechanisms targeted by oral contraceptives is to stop ovulation. Ovulation is the release of eggs from the ovaries of females. Thus, there is no egg for fertilization which prevents pregnancy.
    ● Mechanism 2 is incorrect: Thickening of cervical mucus is another mechanism. This creates a barrier that prevents sperm from entering the uterus and eventually reaching the egg.
    ● Mechanism 3 is incorrect: Thinning of uterus lining is also achieved through oral contraceptive pills. This prevents fertilized eggs from getting attached and growing into a fetus.

    Incorrect

    Answer: A
    Explanation
    The birth control pill is a type of oral contraception that uses hormones to prevent pregnancy. The hormones in birth control pills prevent pregnancy by blocking conception when sperm fertilizes an egg. They also cause changes in the uterus so that it can’t support a pregnancy. Oral contraceptive pills are either combined estrogen-progesterone (also called combined oral contraceptive pill- COC) or progesterone-only pill (POP).
    ● Mechanism 1 is correct: One of the mechanisms targeted by oral contraceptives is to stop ovulation. Ovulation is the release of eggs from the ovaries of females. Thus, there is no egg for fertilization which prevents pregnancy.
    ● Mechanism 2 is incorrect: Thickening of cervical mucus is another mechanism. This creates a barrier that prevents sperm from entering the uterus and eventually reaching the egg.
    ● Mechanism 3 is incorrect: Thinning of uterus lining is also achieved through oral contraceptive pills. This prevents fertilized eggs from getting attached and growing into a fetus.

  2. Question 2 of 5
    2. Question

    2. With reference to the Worldcoin project, consider the following statements:
    1. It aims to create one of the world’s largest cryptocurrency networks for digital finance.
    2. Fingerprint scan will be used for authentication and signup purposes on the network.
    3. It will be open to everyone regardless of his/her country.
    How many of the above statements are correct?

    Correct

    Answer: B
    Explanation
    A project called Worldcoin has been launched by OpenAI, an Artificial Intelligence company. The project claims to be building the world’s largest identity and financial public network.
    ● Statement 1 is correct: The project aims to introduce Worldcoin token as a new cryptocurrency that offers a new and unique method for identification to reduce the risk of fraud. Worldcoin token (WLD) can be used as a medium of purchases and transfers globally using digital assets and traditional currencies.
    ● Statement 2 is incorrect: Iris scan (eye) is essential to sign up to the network. It will be done through a ball-like object called an ‘orb’. Once the orb’s iris scan verifies the person is a real human, it creates a World ID for them. The biometric data would help differentiate between humans and Artificial Intelligence systems and prevent duplication of IDs from the same person. It can then be used as an ID in a variety of everyday applications – such as a cryptocurrency wallet – without revealing the user’s identity.
    ● Statement 3 is correct: The Worldcoin protocol is intended to be the world’s largest identity and financial public network, open to everyone regardless of their country, background, or economic status.

    Incorrect

    Answer: B
    Explanation
    A project called Worldcoin has been launched by OpenAI, an Artificial Intelligence company. The project claims to be building the world’s largest identity and financial public network.
    ● Statement 1 is correct: The project aims to introduce Worldcoin token as a new cryptocurrency that offers a new and unique method for identification to reduce the risk of fraud. Worldcoin token (WLD) can be used as a medium of purchases and transfers globally using digital assets and traditional currencies.
    ● Statement 2 is incorrect: Iris scan (eye) is essential to sign up to the network. It will be done through a ball-like object called an ‘orb’. Once the orb’s iris scan verifies the person is a real human, it creates a World ID for them. The biometric data would help differentiate between humans and Artificial Intelligence systems and prevent duplication of IDs from the same person. It can then be used as an ID in a variety of everyday applications – such as a cryptocurrency wallet – without revealing the user’s identity.
    ● Statement 3 is correct: The Worldcoin protocol is intended to be the world’s largest identity and financial public network, open to everyone regardless of their country, background, or economic status.

  3. Question 3 of 5
    3. Question

    3. With reference to the ‘Formation water’, consider the following statements:
    1. It appears during the mining of coal from underground.
    2. It has a very high salt content.
    3. It cannot be treated by any means.
    How many of the above statements are correct?

    Correct

    Answer: A
    Explanation
    ● Statement 1 is incorrect: Formation water is water that appears during the drilling process for oil and gas extraction. It is very dense and viscous.
    ● Statement 2 is correct: The Formation water consists of oily components, brine solutions, and solvents that are used during various phases in the oil industry. It has a high salt content which includes sodium, potassium, calcium, and magnesium salts.
    ● Statement 3 is incorrect: There are ways and means to treat formation water. It depends on the usage of the treated product water.

    Treatment of the formation water
    The process can be summed up in four stages:
    1. Separation: removing crude, oil and other contaminants in the water that can clog filters downstream.
    2. Suspended Solid Removal: eliminating small size particles remaining.
    3. Dissolved solid removal: using low- and high-pressure membranes.
    4. Conditioning: depending on the quality of the water it may be a post treatment station for disinfection or activated carbon filtration.
    More about formation water
    ● If formation water is released without treatment, it can lead to the degradation of water quality, posing a significant threat to aquatic life.
    ● The toxins in formation water can also bio-magnify into the human food chain, if fish and other aquatic products, harvested from the affected area, are consumed.

    Incorrect

    Answer: A
    Explanation
    ● Statement 1 is incorrect: Formation water is water that appears during the drilling process for oil and gas extraction. It is very dense and viscous.
    ● Statement 2 is correct: The Formation water consists of oily components, brine solutions, and solvents that are used during various phases in the oil industry. It has a high salt content which includes sodium, potassium, calcium, and magnesium salts.
    ● Statement 3 is incorrect: There are ways and means to treat formation water. It depends on the usage of the treated product water.

    Treatment of the formation water
    The process can be summed up in four stages:
    1. Separation: removing crude, oil and other contaminants in the water that can clog filters downstream.
    2. Suspended Solid Removal: eliminating small size particles remaining.
    3. Dissolved solid removal: using low- and high-pressure membranes.
    4. Conditioning: depending on the quality of the water it may be a post treatment station for disinfection or activated carbon filtration.
    More about formation water
    ● If formation water is released without treatment, it can lead to the degradation of water quality, posing a significant threat to aquatic life.
    ● The toxins in formation water can also bio-magnify into the human food chain, if fish and other aquatic products, harvested from the affected area, are consumed.

  4. Question 4 of 5
    4. Question

    4. The ‘Turing Test’ is related to which of the following scientific developments?

    Correct

    Answer: B
    Explanation
    ● The Turing Test is used in artificial intelligence (AI) for determining whether a computer is capable of thinking like a human being. The test is named after Alan Turing, the founder of the Turing Test and an English computer scientist, cryptanalyst, mathematician, and theoretical biologist.
    ● The original Turing Test requires three terminals, each of which is physically separated from the other two. One terminal is operated by a computer, while the other two are operated by humans.
    ● During the test, one of the human being functions as the questioner, while the second human and the computer function as respondents. The questioner interrogates the respondents within a specific subject area, using a specified format and context. After a preset length of time or number of questions, the questioner is then asked to decide which respondent was human and which was a computer.

    Incorrect

    Answer: B
    Explanation
    ● The Turing Test is used in artificial intelligence (AI) for determining whether a computer is capable of thinking like a human being. The test is named after Alan Turing, the founder of the Turing Test and an English computer scientist, cryptanalyst, mathematician, and theoretical biologist.
    ● The original Turing Test requires three terminals, each of which is physically separated from the other two. One terminal is operated by a computer, while the other two are operated by humans.
    ● During the test, one of the human being functions as the questioner, while the second human and the computer function as respondents. The questioner interrogates the respondents within a specific subject area, using a specified format and context. After a preset length of time or number of questions, the questioner is then asked to decide which respondent was human and which was a computer.

  5. Question 5 of 5
    5. Question

    5. Meissner Effect is related to which of the following technologies?

    Correct

    Answer: B
    Explanation
    ● Meissner effect is the expulsion of a magnetic field from the interior of a material that is in the process of becoming a superconductor.

    ● A super conductor loses its resistance to the flow of electrical currents when cooled below a certain temperature called the critical temperature. It is close to absolute zero or zero kelvin.
    ● The Meissner Effect is used in magnetic levitation, which means a body is suspended with no support except a magnetic field. Modern high-speed bullet trains use the phenomenon of magnetic levitation.

    Incorrect

    Answer: B
    Explanation
    ● Meissner effect is the expulsion of a magnetic field from the interior of a material that is in the process of becoming a superconductor.

    ● A super conductor loses its resistance to the flow of electrical currents when cooled below a certain temperature called the critical temperature. It is close to absolute zero or zero kelvin.
    ● The Meissner Effect is used in magnetic levitation, which means a body is suspended with no support except a magnetic field. Modern high-speed bullet trains use the phenomenon of magnetic levitation.

window.wpAdvQuizInitList = window.wpAdvQuizInitList || []; window.wpAdvQuizInitList.push({ id: '#wpAdvQuiz_594', init: { quizId: 594, mode: 0, globalPoints: 10, timelimit: 0, resultsGrade: [0], bo: 0, qpp: 0, catPoints: [10], formPos: 0, lbn: "Finish quiz", json: {"2764":{"type":"single","id":2764,"catId":0,"points":2,"correct":[1,0,0,0]},"2765":{"type":"single","id":2765,"catId":0,"points":2,"correct":[0,1,0,0]},"2766":{"type":"single","id":2766,"catId":0,"points":2,"correct":[1,0,0,0]},"2767":{"type":"single","id":2767,"catId":0,"points":2,"correct":[0,1,0,0]},"2768":{"type":"single","id":2768,"catId":0,"points":2,"correct":[0,1,0,0]}} } });




PANKAJ BANSAL VS UNION OF INDIA: NO RETROSPECTIVE EFFECT

TAG: GS 2 POLITY

CONTEXT:  The Supreme Court (SC) in its recent judgment has ruled that its October 3 judgment, requiring the Enforcement Directorate (ED) to provide a copy of the grounds of arrest at the time of an accused being arrested under money laundering charges, does not apply retrospectively. This new judgement, pronounced by the bench led by J.Bela Trivedi has restricted the scope of Bansal verdict.

EXPLANATION

  • Several petitions have been filed across courts, alleging illegal arrests by the ED on grounds that it only orally intimated grounds of arrest to the accused.
  • The Bench held that non-furnishing of grounds of arrest till the date of pronouncement in Pankaj Bansal case cannot be held to be illegal.
  • The apex court Bench further said that the accused need to be informed of the grounds of the arrest in writing within 24 hours and not necessarily at the time of arrest. But, oral communication is necessary.
  • This, in effect, dilutes its earlier dictum.
  • Provision of written communication about the grounds of arrest within 24 hours of arrest is in compliance with Section 19 of the Prevention of Money Laundering Act, 2002 and also with Article 22 (1) of the Indian constitution.

ED AND ITS POWERS

  • The Directorate of Enforcement is a multi-disciplinary organisation mandated with investigation of offences of money laundering and violations of foreign exchange laws.

The statutory functions of the ED are:

  • The Prevention of Money Laundering Act, 2002 (PMLA):
  • Under this, the ED has been given the responsibility to enforce the provisions of the PMLA by conducting investigation to trace the assets derived from proceeds of crime, to provisionally attach the property and to ensure prosecution of the offenders and confiscation of the property by the Special court.
  • The Foreign Exchange Management Act, 1999 (FEMA):
  • Under this, the ED conducts investigation into suspected contraventions of foreign exchange laws and regulations, to adjudicate and impose penalties on those adjudged to have contravened the law.
  • The Fugitive Economic Offenders Act, 2018 (FEOA):
  • The Directorate is mandated to attach the properties of the fugitive economic offenders who have escaped from the India warranting arrest and provide for the confiscation of their properties to the Central Government.

OUTCOMES OF THE CLARIFICATION

  • After the October 3 judgment, several PMLA undertrials had challenged their arrests based on Bansal’s three principles – first, the non-supply of written grounds; second, the absence of grounds other than mere “evasiveness”; and third, the presence of mala fides.
  • Now, since the judgment cannot be applied retrospectively, those cases will no longer be valid.

CONCLUSION: The Pankaj Bansal judgment was a valuable step in terms of setting institutional accountability of the ED. But the current judgment by the SC seems to dilute the safeguard provided by Bansal case and hence requires reconsideration.

Source : https://www.livelaw.in/supreme-court/ed-can-give-reasons-in-writing-to-accused-within-24-hrs-of-arrest-pankaj-bansal-judgment-doesnt-apply-retrospectively-supreme-court-244662




WHY COP28 COAL BREATHER IS IMPORTANT FOR INDIA?

TAG: GS 3 ECOLOGY AND ENVIRONMENT

CONTEXT: Despite India’s push to increasing green capacity in line with its nationally determined contributions (NDC), the policymakers in India have realised that unless viable energy storage options are available, green energy addition will not make any sense. In view of this, the COP28’s decision to stick to ‘transitioning away from fossil fuel’, and not the complete ‘phase-out’, has come as a breather for India.

EXPLANATION

  • There was a strong pushback by India and countries like China to a proposal stipulating that no new coal-fired power plants can be commissioned without an in-built carbon capture and storage facility.
  • Their opposition, ultimately, led to the omission of the clause from the final climate deal of
  • Recently, the Union Power Ministry has decided to step up its coal-powered generation capacity, with plans to add at least 80 gigawatts (GW, where 1 GW is equal to 1,000 megawatts or MW) by 2031-32.

IS RENEWABLES’ PUSH WANING?

  • As part of its emission-reduction resolve, the government stepped up renewable capacity addition over the last decade.
  • Now, more than 40% of installed electricity capacity comes from non-fossil fuel sources. Solar and wind capacity is now more than 30%.
  • With this, the grid is increasingly powered by renewables now, which creates the problem of intermittency, a significant challenge from a grid management perspective.

WHAT IS ‘INTERMITTENCY’ IN RENEWABLES

  • Renewable energy cannot always consistently produce energy at all hours of the day. Energy production fluctuates with renewable energy. This is termed as Electricity is generated only when the sun shines or when the wind blows, which is not always in sync with the demand cycle.
  • On the other hand, power output generated by fossil fuel-based power plants using coal or natural gas is constant. This has often been referred to as base-load energy.
  • Hence, to counter the problem with renewables, power utilities are forced to keep old thermal units on standby — but that involves high fixed costs payouts.

INDIA’S NEW COAL THRUST

  • Although the plan to increase coal-based capacity is broadly in line with the National Electricity Plan for 2022-27, the new blueprint for the country’s power sector marks a discernible pivot and a policy reversal from earlier years.
  • It underscores the country’s resolve to revert to coal for desperately-needed baseload capacity.

STORAGE CONUNDRUMS

  • Without storage, incremental renewable power capacity poses problems for grid managers.
  • Renewables are not always a viable option for procurers such as state-owned distribution companies (discoms), due to vagaries in the generation trends.
  • It means they have to depend on thermal or nuclear generation to meet the base load demand.
  • Storage is expensive currently, and the Lithium-ion storage battery option for grid application is being ruled out as unviable.

WHAT IS THE WAY OUT?

  • Hydrogen and hybrid generation models blended with off-stream pumped storage are being cited as two alternatives.
  • The government is also looking to mandate renewable energy projects with more than 5-megawatt (MW) capacity to install energy storage systems (ESS).
  • However, there is a recognition that coal-fired capacity needs to stay as base load for the time being.
  • More steps are required to be taken at the policy and regulatory levels for ensuring an enabling ecosystem for storage systems.

CONCLUSION: While cutting down on fossil fuel usage is imperative to meet the Paris Climate Agreement goals, access to viable storage technology is equally important. The breather may have been given, but the developed countries need to accelerate their mitigation efforts to allow a fair share of remaining carbon space to be available to developing countries.

Source: https://indianexpress.com/article/explained/explained-economics/india-coal-renewable-energy-storage-9069676/




CONSEQUENCES OF HOTTEST ARCTIC SUMMER

TAG: GS 3: ECOLOGY AND ENVIRONMENT

THE CONTEXT: The 2023 summer in the Arctic was the warmest on record and the region has been warming nearly four times faster than the global average since 1979. This has been highlighted by the National Oceanic and Atmospheric Administration’s (NOAA) annual Arctic Report Card, a peer-reviewed analysis conducted by 82 scientists from 13 countries.

EXPLANATION:

The report has highlighted 5 major consequences of the rising temperatures in the Arctic region:

  1. Thawing Of Subsea Permafrost
  2. Food Insecurity
  3. Raging Wildfires
  4. Severe Flooding
  5. Greenland Ice Sheet Melting

THAWING OF SUBSEA PERMAFROST

As subsea permafrost thaws, the organic matter within it begins to decay. This decomposition process releases large amounts of methane and carbon dioxide into the surrounding water and atmosphere.

Impact

  • Global Warming: The released greenhouse gases trap heat in the Earth’s atmosphere, contributing to the ongoing trend of global warming.
  • Ocean Acidification: Carbon dioxide also leads to ocean acidification, lowering the pH of the ocean. This can have detrimental effects on marine life, particularly organisms with calcium carbonate shells or skeletons, such as corals, mollusks, and certain plankton species.

FOOD INSECURITY

Salmon is a crucial part of the traditional diet in many Indigenous cultures, providing essential nutrients and serving as a staple food source.

Impact

  • Food crisis: The reduced numbers and size of Chinook and chum salmon represent a significant threat to the food security of Indigenous communities.
  • Cultural impact: The decline in salmon populations disrupts the cultural fabric of these communities, impacting their way of life and connection to the land.
  • Increase in conflicts: This can also result in conflicts over access to dwindling salmon resources, further straining the relationships between different user groups.

RAGING WILDFIRES

High temperatures and below-average rainfall created favorable conditions for increased instances of wildfires. Canada experienced its worst wildfire season, with over 10 million acres burned in the Northwest Territories.

Impact

  • Destruction of Ecosystems: Wildfires lead to the rapid destruction of vegetation and ecosystems which have long-lasting effects on the local flora and fauna.
  • Air Quality: The smoke produced by wildfires can travel over long distances, affecting air quality. This poses serious health risks, particularly for individuals with respiratory conditions.
  • Evacuations and Displacement: The need to evacuate two-thirds of the population in the affected territories disrupts communities and places a strain on emergency services.

SEVERE FLOODING

Glacier meltwater caused floods, leading to property damage, and highlighting the tangible consequences of glacial retreat.

Impact

  • Property Damage: Flooding, especially when caused by the meltwater from glaciers, can lead to severe damage to infrastructure, homes, and businesses.
  • Risk to Lives: Flooding poses a direct threat to human lives. The sudden release of water from a glacial lake can lead to flash floods, putting communities at risk.
  • Displacement and Community Disruption: Flooding can force communities to evacuate temporarily or permanently which can weaken the strong community bonding.

GREENLAND ICE SHEET MELTING

The melting of the Greenland Ice Sheet has far-reaching consequences, affecting both local and global environments.

Impact

  • Sea Level Rise: The Greenland Ice Sheet continues to lose mass, contributing to rising sea levels. Sea level rise poses a global threat to coastal communities and ecosystems.
  • Global Climate System: The melting of large ice sheets can disrupt ocean currents and impact global climate systems.
  • Local Environmental Changes: The melting of ice alters the local environment in Greenland. This includes changes in the availability of freshwater, affecting ecosystems and wildlife.

CONCLUSION

  • Arctic’s vulnerability underscores the pressing need for concerted efforts to combat climate change.
  • The report highlights the far-reaching consequences of global warming, urging society to take collective action to mitigate these impacts and build resilience in the face of a changing climate.

SOURCE: https://indianexpress.com/article/explained/explained-climate/arctic-hottest-summer-climate-change-9069590/




SECOND AVATAR OF THE CRIMINAL LAW BILLS: THE KEY CHANGES

TAG: GS 2: POLITY AND GOVERNANCE

THE CONTEXT: The three Bills introduced by the Union government in the Lok Sabha in August 2023 to replace respectively the Indian Penal Code, the Criminal Procedure Code and the Indian Evidence Act were unexpectedly withdrawn on December 9, 2023. They were replaced by revised second drafts which were tabled by the Union home minister on December 12, 2023 in the Lok Sabha. 
EXPLANATION:

BACKGROUND:

  • The August 2023 first drafts of the three Bills had drawn wide public outrage as an assault on our democracy.
  • These were reviewed by the Parliamentary Standing Committee, reflecting the extremely poor quality and the alarming nature of the Bills, it was constrained to recommend a slew of changes. The Committee went to the extent of cautiously expressing mild concern about the vagueness of crucial definitions in the drafts
  • In their second drafts too, the character of the Bills remains fundamentally anti-democratic. The fundamental objection to the first draft of the Bills is not removed by the second draft.

Key Modifications proposed in drafts are:

1. The most important change which will replace the Indian Penal Code, is that the government has entirely pulled back from the first draft’s expansion of the crime of terrorism beyond the existing definition in the Unlawful Activities (Prevention) Act, 1967 (UAPA).

  • The UAPA defines as terrorist any act “with intent to threaten or likely to threaten the unity, integrity, security economic security, or sovereignty of India or with intent to strike terror or likely to strike terror in the people or any section of the people in India or in any foreign country”.
  • Definition in the first BNS draft included as terrorism extremely vague acts such as “intimidating the general public or a segment thereof”, “disturbing public order”, “creating an atmosphere or spreading a message of fear”; “destabilising or destroying the political, economic, or social structures of the country”, or “creating a public emergency or undermining public safety”.
  • Under the first BNS draft, these acts would be terrorist acts even if they are in the form of mere non-violent speech that does not involve the commission of any crime. Each of these vague formulations has the potential to be abused to lock up virtually anyone by converting legitimate public discourse into ‘terrorism’.
  • The second draft of BNS withdraws the definition of a terrorist act in the first draft and entirely adopts the UAPA definition. There is one exception UAPA includes in terrorism the “production or smuggling or circulation only of high quality counterfeit Indian paper currency, coin or of any other material”. Whereas the second BNS draft widens this definition to cover the same activities with respect to any counterfeit Indian paper currency, coin or of any other material.
  • The second draft of BNS also changes the punishment for ‘terrorism which results in death’ from life imprisonment without parole to life imprisonment as provided in UAPA (i.e., without ruling out parole).

Criticism:

  • Double-barrelled weapon: While the modifications to the BNS definition of terrorism are welcome, it is disappointing that the government refused to drop the terrorism offence altogether from BNS as it is already covered under UAPA.  With this new BNS provision, the government will now have a double-barrelled weapon to prosecute and imprison terrorism under two statutes – a special law (UAPA) and the other a general law (BNS).
  • Extraordinary police discretion: Given that one statute (UAPA) has some safeguards and a special court and the other (BNS) does not, this in itself creates an opportunity for potential rent-seeking and corruption on the exercise of this extraordinary police discretion. No justification has been provided for the need to maintain the offence of terrorism in two separate statutes.

2. Another potent weapon to be misused against non-violent dissent lay in the vague definition of “petty organised crime” in the first BNS draft. Under which any crime that causes general feelings of insecurity among citizens relating to thirteen enumerated acts and “other common forms of organised crime committed by organised criminal groups or gangs” was criminalised.

The government has pulled back on this open-ended definition and replaced it in the second draft with a more circumscribed definition: “Whoever, being a member of a group or gang, either singly or jointly, commits any act of theft, snatching, cheating, unauthorised selling of tickets, unauthorised betting or gambling, selling of public examination question papers or any other similar criminal act, is said to commit petty organised crime.”

3. Punishments

  • In the second BNS draft, the government dropped the earlier proposal that life imprisonment in all cases shall be “imprisonment for remainder of a person’s natural life”.
  • The second BNS draft clarifies punishment for ‘culpable homicide not amounting to murder’ which had not been clearly drafted in the first draft.
  • Under the revised version, causing death by any rash or negligent act not amounting to culpable homicide will be punishable by five years’ imprisonment, reduced from seven years.

4. Mental unsoundness 

  • The first drafts of the Bills wrongly substituted the IPC, CrPC and Evidence Act concept of “mental unsoundness” with the term “mental illness” without noticing the critical distinction between these two concepts.
  • This error has been rectified in the second draft of all three Bills.

5. Use of electronic technology

  • The draft Bills clearly indicate a lack of clarity and understanding about the use of electronic means in criminal justice.
  • The second BNSS draft deletes various proceedings that were authorised to be conducted electronically in the first draft without explaining the need for the change.

Some key recommendations not accepted

  • The Standing Committee’s recommendations that
  • non-consensual sexual acts covered by IPC Section 377 should continue to be criminalized
  • grounds must be provided by the Executive for commutation of sentences
  • a special provision be included to protect healthcare workers
  • adultery be maintained as a gender-neutral crime.

Criticisms of the Bills:

  • The Bills weaponise the police and the criminal justice system to give the political leadership at all levels centre, state and local. It gives greater opportunity to abuse the criminal justice system for political gain through selective, targeted and politically biased prosecution against ideological and political rivals.
  • The Bills create and maintain high-sounding but vaguely worded and easily abused offences such as “terrorism”; “organised crime”; endangering “sovereignty, unity and integrity of India”; and what we may call “sedition plus”.
  • The Bills scale up police powers and discretion in a number of areas to be used for prosecuting these vague crimes. Violating well-established judicial standards, the Bills dilute the legal obligation of the police to file FIRs and register cases by providing police the option to conduct a preliminary inquiry before registering an FIR in certain cases.
  • To expand the surveillance state, the Bills mandate the provision of biometrics by those who are not accused but are arrested in a case enhancing the incentive for arresting a person who is sought to be surveilled.
  • There is no real effort in the Bills to enhance police accountability to the people. At a time when the judiciary is unable to protect citizens from malicious and selective prosecution. These Bills enhance repression in our country.

CONCLUSION: Rather than strengthening criminal justice administration, the Bills primarily serve political aims.  Like many other historic pieces of legislation, these three pivotal laws that will deeply affect the lives of all Indians will also be rushed through parliament with the explicit aim of avoiding any meaningful debate or genuine discussion on them.

Source: https://thewire.in/government/second-avatar-criminal-law-bills-has-anything-changed




MALDIVES TO NOT RENEW 2019 WATER SURVEY PACT WITH INDIA

TAG: GS 2: INTERNATIONAL RELATIONS

THE CONTEXT: The Maldives government has decided to not renew an agreement with India that allowed India to conduct hydrographic surveys in Maldivian waters.

EXPLANATION:

  • The agreement was signed in 2019 during Indian Prime Minister’s visit to the islands. Various MoUs were signed during the visit, including one for Cooperation in the Field of Hydrography between the Indian Navy and the Maldives National Defence Force (MNDF).
  • The new government of Maldives had earlier requested that India should pull out its military personnel deployed in the country.

Hydrographic survey:

  1. Map out water depth
  2. Shape of the seafloor and coastline
  3. Location of possible obstructions
  4. Physical features of water bodies to ensure the efficiency and safety of maritime transportation.

Hydrographic survey pact:

  • The first meeting of the Joint Commission on Hydrography was held in the Maldives in September 2019.
  • So far, three joint hydrographic surveys have been undertaken – in 2021, 2022, and 2023.
  • The Indian Naval Ship (INS) Darshak carried out the first Joint Hydrographic survey in February and March of 2021.
  • The second Joint Hydrographic survey was carried out by the INS Sutlej from April 18 to May 24, 2022. During the period, the ship surveyed Northern Maldives and the Male area including Thilafushi, Hulhumale and Male port on the request of the Government of Maldives.
  • Multi-beam echo sounders were used. The surveys were done to generate updated Navigational Charts/ Electronic Navigational Charts of the areas, which would help sectors such as Tourism, Fisheries, Agriculture, etc.
  • Training was also imparted to MNDF personnel on the use of survey equipment. India said it would train more MNDF personnel at its Hydrographic Institute in Goa.
  • The third survey was conducted between January and February 2023, by INS Investigator. It identified 52 new shoals, which are emerging ridge-like natural structures in the sea, within the Ihavandhippolhu atoll.

Water survey pacts with other countries:

  • India’s oldest Hydrographic Survey ship, INS Sandhayak, was decommissioned in 2021.

It undertook more than 200 major hydrographic surveys along the Western and Eastern coasts of the Indian peninsula, and the Andaman Sea, as well as surveys in neighbouring countries including Sri Lanka, Myanmar, and Bangladesh.

  • The government has previously cited a UN study that says approximately 50 per cent of coastal states have no hydrographic capability and another 25 per cent have only limited capabilities. Only the remaining 25 per cent, including India, have adequate hydrographic capabilities.
  • National Hydrographic Office’s website states “There is, immense scope for international co-operation in hydrography, particularly, in Asia and Africa, where 36% and 64% of the waters respectively, are yet to be surveyed systematically.
  • It says Indian survey ships have assisted Kenya, Mauritius, Mozambique, Maldives, Oman, Seychelles, Sri Lanka and Tanzania in the past.

So why does Maldives want to end the pact?

  • Change of regime: There is change in regime in the country following the upcoming elections. Earlier President of the Maldivian Democratic Party (MDP), who was in power from 2018 to 2023, was seen as being more favourable to India, but his successor of the Progressive Party of Maldives (PPM) is being seen as more pro-China.
  • China’s influence: While the Maldives has traditionally been a part of India’s sphere of influence. In recent decades China has sought to project its power aggressively in the Indian Ocean, including through massive investments in infrastructure projects as Belt and Road Initiative (BRI).
  • Self interest: Maldivian administration believes it is “best for national security to improve the Maldivian military’s capacity to conduct such surveys, and protect such sensitive information”.

Source: https://indianexpress.com/article/explained/everyday-explainers/maldives-survey-pact-india-hydrographic-explained-9069657/




ACCELERATION FORETOLD: ON VOLATILE FOOD PRICES

THE CONTEXT: There is resurgence in headline retail inflation in November which was totally unexpected after the RBI just predicted a  small increase. It is a stark reminder of the risks volatile food prices pose.

INFLATION TREND ANALYSIS

  • National Statistical Office’s provisional reading of headline inflation shows the Consumer Price Index rose by 5.55% year-on-year to a three-month high, from October’s 4.87%.
  • Food price gains measured by the Consumer Food Price Index accelerated by a steep 209 basis points to 8.7% last month. Cereals and vegetables surged 10.3% and 17.7% inflation, respectively.
  • Vegetable price’s rate surging by almost 15 percentage points from October’s 2.8%. Only potato prices, which continued to remain in deflationary territory, offered some respite.
  • Pulses and sugar are other areas of concern, with the first witnessing more than 20% inflation and the sweetener also experiencing an uptick in the pace of price gains to 6.55%.
  • With the RBI having opted to refrain from raising rates for now, the onus lies on the government to help temper inflation.

ABOUT INFLATION:

  • Inflation is defined by the International Monetary Fund as the rate of increase in prices over a given period, encompassing a broad measure of overall price increase or for specific goods and services.
  • It reflects the rising cost of living and indicates how much more expensive a set of goods or services has become over a specified period, usually a year.

Headline Inflation

  • Headline inflation is the raw inflation figure reported through the Consumer Price Index (CPI) that is released monthly by the Bureau of Labor Statistics.
  • Headline inflation is not adjusted to remove highly volatile figures, including those that can shift regardless of economic conditions.

Core Inflation

  • Core inflation is the change in the costs of goods and services but does not include those from the food and energy sectors.
  • It is most often calculated using the consumer price index (CPI), which is a measure of prices for goods and services.

CAUSES OF INFLATION:

  • Supply Shocks: Inflation is caused due to sudden and unexpected disruption to the supply of goods and services. Some of the reasons for reduction in supply are Natural disasters, geopolitical events, or other unforeseen circumstances.
  • Demand-Pull Inflation: It occurs when the demand for goods and services exceeds their supply. When the overall demand in the economy is high, consumers are willing to pay more for the available goods and services that leads to a general rise in prices.
  • Cost-Push Inflation: It is driven by an increase in the production costs for goods and services. This can be caused by factors such as increased incomes, increased costs of raw materials, or disruptions in the supply chain.
  • Increase in the money supply in an economy: When there is more money in circulation, consumers have more purchasing power, which can drive up demand and prices.

CONCERNS RELATED TO INFLATION:

  • Decreased Purchasing Power: Inflation erodes the purchasing power of money, meaning that with the same amount of money, individuals can buy fewer goods and services.
  • Uncertainty and Planning Challenges: High inflation can create uncertainty in the economy. It becomes challenging to plan for the future when prices are constantly changing. Long- planning term becomes difficult, and uncertainty can lead to hesitancy in making investment decisions. This forces the government to spur the investments and leads to crowding-out effects.
  • Reduces overall demand: The eventual fallout of reduced purchasing power is that consumers demand fewer goods and services.
  • Worsens the exchange rate: High inflation means the rupee is losing its power. Investors will take away their capital because of reduced returns. Thus, high inflation can lead to worsening of exchange rate.

THE WAY FORWARD:

  • Monetary Policy: The Reserve Bank of India (RBI), India’s central bank, plays a crucial role in controlling inflation through monetary policy. The RBI adjusts key interest rates, such as the repo rate, to influence money supply and credit in the economy. These monetary measures can help in tackling inflation.
  • Fiscal Policy Measures: The government uses fiscal policies like taxation and public spending to manage inflation. Appropriate fiscal measures can help in curbing demand and controlling inflationary pressures. Higher taxes can reduce disposable income, curbing spending and inflation.
  • Food Price Management: Given that food prices often contribute significantly to inflation in India, the government needs to implement initiatives to manage food supplies and prices. For examples, there is need to strengthening Minimum Support Price (MSP) and the Public Distribution System (PDS). To prevent artificial scarcity and price manipulation, the government need to conduct regular checks  against hoarding and black marketing.

THE CONCLUSION:

There is a need to address the high commodity prices and shortages of raw materials to support the consumption in the country by preserving macro-financial stability.

PREVIOUS YEAR QUESTIONS

Q.1 Do you agree with the view that steady GDP growth and low inflation have left the Indian economy in good shape? Give reasons in support of your arguments.(2019)

Q.2 It is argued that the strategy of inclusive growth is intended to meet the objectives of inclusiveness and sustainability together. Comment on this statement.(2019)

MAINS PRACTICE QUESTION

Q.1 How does inflation affect the consumption and economic growth in the country?. Suggest measures to tackle high inflation in India.

SOURCE: https://www.thehindu.com/opinion/editorial/acceleration-foretold-on-volatile-food-prices/article67642176.ece




Day-554 | Daily MCQs | UPSC Prelims | POLITY

Day-554

Time limit: 0

Quiz-summary

0 of 5 questions completed

Questions:

  1. 1
  2. 2
  3. 3
  4. 4
  5. 5

Information

DAILY MCQ

You have already completed the quiz before. Hence you can not start it again.

Quiz is loading...

You must sign in or sign up to start the quiz.

You have to finish following quiz, to start this quiz:

Results

0 of 5 questions answered correctly

Your time:

Time has elapsed

You have reached 0 of 0 points, (0)

Categories

  1. Not categorized 0%
  1. 1
  2. 2
  3. 3
  4. 4
  5. 5
  1. Answered
  2. Review
  1. Question 1 of 5
    1. Question

    1. Consider the following:
    1. Consolidated Fund of India
    2. Contingency Fund of India
    3. Public Account of India
    The Union Government Finance Accounts depicts the receipts and payments from how many of the funds mentioned above?

    Correct

    Answer: C
    Explanation:
    Statement 1 is correct: The Annual Accounts of the Union Government provide a comprehensive overview of the government’s financial transactions for a given fiscal year. These audit of these accounts are presented to the Parliament by the President of India while the report is prepared by the Comptroller and Auditor General (CAG) of India, typically within six months of the conclusion of the fiscal year. The Annual Accounts serve as a crucial tool for ensuring transparency and accountability in the government’s financial management.
    Components of the Annual Accounts: It consists of two main components:
    1. Finance Accounts: The Finance Accounts provide a detailed record of all receipts and payments made by the government during the fiscal year. These accounts include information on revenue collection, expenditure incurred; from the Consolidated Fund of India, Contingency Fund and Public Account; and the overall financial position of the government.(Hence Option C is correct)
    2. Appropriation Accounts: The Appropriation Accounts compare the actual expenditure incurred by the government with the amounts authorized by the Parliament through the annual budget. These accounts highlight any variances between the budgeted and actual expenditure and provide explanations for these variations.

    Incorrect

    Answer: C
    Explanation:
    Statement 1 is correct: The Annual Accounts of the Union Government provide a comprehensive overview of the government’s financial transactions for a given fiscal year. These audit of these accounts are presented to the Parliament by the President of India while the report is prepared by the Comptroller and Auditor General (CAG) of India, typically within six months of the conclusion of the fiscal year. The Annual Accounts serve as a crucial tool for ensuring transparency and accountability in the government’s financial management.
    Components of the Annual Accounts: It consists of two main components:
    1. Finance Accounts: The Finance Accounts provide a detailed record of all receipts and payments made by the government during the fiscal year. These accounts include information on revenue collection, expenditure incurred; from the Consolidated Fund of India, Contingency Fund and Public Account; and the overall financial position of the government.(Hence Option C is correct)
    2. Appropriation Accounts: The Appropriation Accounts compare the actual expenditure incurred by the government with the amounts authorized by the Parliament through the annual budget. These accounts highlight any variances between the budgeted and actual expenditure and provide explanations for these variations.

  2. Question 2 of 5
    2. Question

    2. Consider the following:
    1. Equality before the law
    2. Transparency and predictability
    3. Civic duty
    4. Independence of the judiciary
    How many of the given above are considered as the important features of “Rule of Law” in the Indian context?

    Correct

    Answer: C
    Explanation:
    Option 1 is correct: The rule of law implies that everyone, regardless of their status or position, is subject to the law and must abide by it. This means that no one is above the law and that everyone is treated equally under the law.The principle of equality before the law is essential for ensuring fairness and justice in society. It prevents discrimination and ensures that everyone has equal access to the legal system. When the law is applied equally, it creates a sense of trust and legitimacy among the people, fostering a more stable and harmonious society.
    Option 2 is correct: The rule of law requires that the law be clear, transparent, and predictable. This means that the law should be written in a way that is easy to understand and that it should be applied consistently and impartially.When the law is clear and predictable, it reduces uncertainty and promotes compliance. It also allows individuals to hold the government accountable for its actions and to challenge decisions that they believe violate their rights.
    Option 3 is incorrect: Civic duties are the responsibilities that citizens have to their communities and their country. These duties are often enshrined in law, but they can also be based on tradition, custom, or moral values. In India, there are a number of civic duties that are considered to be important for all citizens. These duties include:
    ● Obeying the law: This is the most basic civic duty, and it is essential for maintaining order and stability in society. Citizens should obey all laws, even those they disagree with, and they should report any violations to the authorities.
    ● Voting: Voting is another fundamental civic duty. It allows citizens to participate in the democratic process and to choose their representatives. Citizens should register to vote and participate in all elections.
    ● Paying taxes: Taxes are an important source of revenue for the government, and they are used to fund a variety of public services, such as education, healthcare, and infrastructure. Citizens should pay their taxes on time and in full.
    ● Protecting the environment: The environment is a vital resource that we must protect for future generations. Citizens should take steps to conserve resources, reduce pollution, and recycle.
    In addition to these general civic duties, there are also a number of specific civic duties that are enshrined in the Indian Constitution. These duties include:
    ● Abide by the Constitution and respect its ideals and institutions, the National Flag and the National Anthem: This duty requires citizens to uphold the values and principles of the Constitution and to respect the symbols of national unity.
    ● Cherish and follow the noble ideals which inspired our national struggle for freedom: This duty requires citizens to appreciate the sacrifices made by those who fought for India’s independence and to strive to live up to their ideals.
    ● Uphold and protect the sovereignty, unity, and integrity of India: This duty requires citizens to defend India’s borders, to promote unity among its people, and to protect its national interests.
    Option 4 is correct: The rule of law requires that the judiciary be independent of the executive and legislative branches of government. This means that the judiciary should be able to make decisions without fear of political interference or intimidation.An independent judiciary is essential for upholding the rule of law and protecting individual rights. When the judiciary is free from political influence, it can ensure that the law is applied fairly and impartially. It can also protect individuals from arbitrary or unjust actions by the government.
    Additional Information:
    Rule of Law vs. Due Process of Law
    While the rule of law and due process of law are related concepts, they are not the same. The rule of law is a broader principle that encompasses the entire legal system, including the laws themselves, the institutions that enforce them, and the values that underpin them.
    Due process of law, on the other hand, is a specific legal concept that refers to the procedures that must be followed when the government takes action against an individual.
    In essence, the rule of law is the foundation upon which due process of law is built. The rule of law ensures that the legal system is fair, just, and impartial, while due process of law ensures that individuals are treated fairly and have their rights protected when the government takes action against them.

    Incorrect

    Answer: C
    Explanation:
    Option 1 is correct: The rule of law implies that everyone, regardless of their status or position, is subject to the law and must abide by it. This means that no one is above the law and that everyone is treated equally under the law.The principle of equality before the law is essential for ensuring fairness and justice in society. It prevents discrimination and ensures that everyone has equal access to the legal system. When the law is applied equally, it creates a sense of trust and legitimacy among the people, fostering a more stable and harmonious society.
    Option 2 is correct: The rule of law requires that the law be clear, transparent, and predictable. This means that the law should be written in a way that is easy to understand and that it should be applied consistently and impartially.When the law is clear and predictable, it reduces uncertainty and promotes compliance. It also allows individuals to hold the government accountable for its actions and to challenge decisions that they believe violate their rights.
    Option 3 is incorrect: Civic duties are the responsibilities that citizens have to their communities and their country. These duties are often enshrined in law, but they can also be based on tradition, custom, or moral values. In India, there are a number of civic duties that are considered to be important for all citizens. These duties include:
    ● Obeying the law: This is the most basic civic duty, and it is essential for maintaining order and stability in society. Citizens should obey all laws, even those they disagree with, and they should report any violations to the authorities.
    ● Voting: Voting is another fundamental civic duty. It allows citizens to participate in the democratic process and to choose their representatives. Citizens should register to vote and participate in all elections.
    ● Paying taxes: Taxes are an important source of revenue for the government, and they are used to fund a variety of public services, such as education, healthcare, and infrastructure. Citizens should pay their taxes on time and in full.
    ● Protecting the environment: The environment is a vital resource that we must protect for future generations. Citizens should take steps to conserve resources, reduce pollution, and recycle.
    In addition to these general civic duties, there are also a number of specific civic duties that are enshrined in the Indian Constitution. These duties include:
    ● Abide by the Constitution and respect its ideals and institutions, the National Flag and the National Anthem: This duty requires citizens to uphold the values and principles of the Constitution and to respect the symbols of national unity.
    ● Cherish and follow the noble ideals which inspired our national struggle for freedom: This duty requires citizens to appreciate the sacrifices made by those who fought for India’s independence and to strive to live up to their ideals.
    ● Uphold and protect the sovereignty, unity, and integrity of India: This duty requires citizens to defend India’s borders, to promote unity among its people, and to protect its national interests.
    Option 4 is correct: The rule of law requires that the judiciary be independent of the executive and legislative branches of government. This means that the judiciary should be able to make decisions without fear of political interference or intimidation.An independent judiciary is essential for upholding the rule of law and protecting individual rights. When the judiciary is free from political influence, it can ensure that the law is applied fairly and impartially. It can also protect individuals from arbitrary or unjust actions by the government.
    Additional Information:
    Rule of Law vs. Due Process of Law
    While the rule of law and due process of law are related concepts, they are not the same. The rule of law is a broader principle that encompasses the entire legal system, including the laws themselves, the institutions that enforce them, and the values that underpin them.
    Due process of law, on the other hand, is a specific legal concept that refers to the procedures that must be followed when the government takes action against an individual.
    In essence, the rule of law is the foundation upon which due process of law is built. The rule of law ensures that the legal system is fair, just, and impartial, while due process of law ensures that individuals are treated fairly and have their rights protected when the government takes action against them.

  3. Question 3 of 5
    3. Question

    3. Consider the following statements about the Mayoral System in India:
    1. Mayoral system in India is not explicitly mentioned in the Constitution of India.
    2. The Mayor is chosen through indirect election by the councilors from among themselves in all the states.
    3. It aims to decentralize the power from the state government to the municipal level
    How many of the above given statements are correct?

    Correct

    Answer: B
    Explanation:
    Statement 1 is correct:The Constitution of India does not explicitly mandate the mayoral system, but it allows states to adopt it through their own legislation. Several Indian states have implemented the mayoral system, including Andhra Pradesh, Chhattisgarh, Jharkhand, Madhya Pradesh, Odisha, Tamil Nadu, and Telangana.The mayor is merely a ceremonial authority, and executive decisions are carried out by the municipal commissioner appointed by the state government.
    Statement 2 is incorrect:The Mayor in the Municipal Corporation is usually chosen through indirect election by the councilors from among themselves. However, few states including Uttarakhand, Chhattisgarh, Jharkhand, Madhya Pradesh, Uttar Pradesh, and Tamil Nadu – have mayors who are elected directly. This inconsistency is because of the lack of clarity about the Mayoral system in the Constitution of India. Also the 74th amendment did not prescribe the manner of election, tenure, or powers of the Mayors/Chairpersons of Urban Local Bodies.
    Statement 3 is correct: The mayoral system aims to decentralize power from the state government to the municipal level, empowering local bodies to make decisions that directly impact their citizens.The mayoral system seeks to bring governance closer to the people, giving them a direct say in matters that affect their daily lives. This decentralization of power enhances local accountability and encourages greater citizen participation in the decision-making process.

    Roles, Functions, and Powers of the Mayor
    The specific roles, functions, and powers of the mayor vary from state to state. However, in general, the mayor is responsible for:
    ● Providing leadership and direction to the municipal administration
    ● Presiding over the meetings of the municipal corporation
    ● Overseeing the implementation of municipal policies and programs
    ● Representing the municipality in official engagements
    ● Ensuring the financial well-being of the municipality
    ● Promoting the interests of the city and its citizens
    The exact extent of the mayor’s powers depends on the specific provisions of the state legislation governing the mayoral system.

    Incorrect

    Answer: B
    Explanation:
    Statement 1 is correct:The Constitution of India does not explicitly mandate the mayoral system, but it allows states to adopt it through their own legislation. Several Indian states have implemented the mayoral system, including Andhra Pradesh, Chhattisgarh, Jharkhand, Madhya Pradesh, Odisha, Tamil Nadu, and Telangana.The mayor is merely a ceremonial authority, and executive decisions are carried out by the municipal commissioner appointed by the state government.
    Statement 2 is incorrect:The Mayor in the Municipal Corporation is usually chosen through indirect election by the councilors from among themselves. However, few states including Uttarakhand, Chhattisgarh, Jharkhand, Madhya Pradesh, Uttar Pradesh, and Tamil Nadu – have mayors who are elected directly. This inconsistency is because of the lack of clarity about the Mayoral system in the Constitution of India. Also the 74th amendment did not prescribe the manner of election, tenure, or powers of the Mayors/Chairpersons of Urban Local Bodies.
    Statement 3 is correct: The mayoral system aims to decentralize power from the state government to the municipal level, empowering local bodies to make decisions that directly impact their citizens.The mayoral system seeks to bring governance closer to the people, giving them a direct say in matters that affect their daily lives. This decentralization of power enhances local accountability and encourages greater citizen participation in the decision-making process.

    Roles, Functions, and Powers of the Mayor
    The specific roles, functions, and powers of the mayor vary from state to state. However, in general, the mayor is responsible for:
    ● Providing leadership and direction to the municipal administration
    ● Presiding over the meetings of the municipal corporation
    ● Overseeing the implementation of municipal policies and programs
    ● Representing the municipality in official engagements
    ● Ensuring the financial well-being of the municipality
    ● Promoting the interests of the city and its citizens
    The exact extent of the mayor’s powers depends on the specific provisions of the state legislation governing the mayoral system.

  4. Question 4 of 5
    4. Question

    4. “Appointment of Governors shall be made on the recommendation of a committee comprising the Prime Minister, the Home Minister, the Lok Sabha Speaker and the concerned Chief Minister of the State”.
    Which of the following commissions made the recommendation given above?

    Correct

    Answer: C
    Explanation: The correct answer is MM Punchhi Commission. Among other recommendations, the recommendations with respect to the appointment and removal of the Governor of the State include:
    ● There should be a say of the state’s Chief Minister while making the Governor’s appointment.
    ● A committee should be formed that is entrusted with the task of appointment of governors. This committee may comprise the Prime Minister, the Home Minister, the Lok Sabha’s speaker and the concerned Chief Minister of the State.
    ● The term of appointment should be five years.
    ● Governor could only be removed via a resolution by the State Legislature.
    Rajamannar Committee (1971) stressed that the Governor of the state should not consider himself as an agent of the centre but play his role as the constitutional head of the State.
    Sarkaria Commission:
    ● The Constitution itself should lay down the process for consulting the chief minister when appointing a state governor.
    ● As long as the council of ministers has a majority in the assembly, the governor cannot dismiss it.
    ● Unless there are some exceptionally compelling circumstances, a governor’s five-year governorship should not be disrupted.

    Incorrect

    Answer: C
    Explanation: The correct answer is MM Punchhi Commission. Among other recommendations, the recommendations with respect to the appointment and removal of the Governor of the State include:
    ● There should be a say of the state’s Chief Minister while making the Governor’s appointment.
    ● A committee should be formed that is entrusted with the task of appointment of governors. This committee may comprise the Prime Minister, the Home Minister, the Lok Sabha’s speaker and the concerned Chief Minister of the State.
    ● The term of appointment should be five years.
    ● Governor could only be removed via a resolution by the State Legislature.
    Rajamannar Committee (1971) stressed that the Governor of the state should not consider himself as an agent of the centre but play his role as the constitutional head of the State.
    Sarkaria Commission:
    ● The Constitution itself should lay down the process for consulting the chief minister when appointing a state governor.
    ● As long as the council of ministers has a majority in the assembly, the governor cannot dismiss it.
    ● Unless there are some exceptionally compelling circumstances, a governor’s five-year governorship should not be disrupted.

  5. Question 5 of 5
    5. Question

    5. The Simon Commission was appointed by the British government to report on the working of which of the following?

    Correct

    Answer: D
    Explanation:
    Government of India Act, 1919 provided for the appointment of the statutory commission to inquire into and report on its working after ten years of coming into force.
    The Government of India Act 1919 was the codified version of the Montague-Chelmsford reforms – named after Edwin Charles Montague and Lord Chelmsford, who held positions of the Secretary of State and Viceroy of British India respectively. The Act was sold to the Indians as ‘a step in the progressive realisation of responsible government in India as an integral part of the empire’.
    The persistent demand for further reforms led the British government to appoint Simon Commission in 1927 and report on the working of the Indian Constitution as established under Government of India Act, 1919.

    Incorrect

    Answer: D
    Explanation:
    Government of India Act, 1919 provided for the appointment of the statutory commission to inquire into and report on its working after ten years of coming into force.
    The Government of India Act 1919 was the codified version of the Montague-Chelmsford reforms – named after Edwin Charles Montague and Lord Chelmsford, who held positions of the Secretary of State and Viceroy of British India respectively. The Act was sold to the Indians as ‘a step in the progressive realisation of responsible government in India as an integral part of the empire’.
    The persistent demand for further reforms led the British government to appoint Simon Commission in 1927 and report on the working of the Indian Constitution as established under Government of India Act, 1919.

window.wpAdvQuizInitList = window.wpAdvQuizInitList || []; window.wpAdvQuizInitList.push({ id: '#wpAdvQuiz_593', init: { quizId: 593, mode: 0, globalPoints: 10, timelimit: 0, resultsGrade: [0], bo: 0, qpp: 0, catPoints: [10], formPos: 0, lbn: "Finish quiz", json: {"2759":{"type":"single","id":2759,"catId":0,"points":2,"correct":[0,0,1,0]},"2760":{"type":"single","id":2760,"catId":0,"points":2,"correct":[0,0,1,0]},"2761":{"type":"single","id":2761,"catId":0,"points":2,"correct":[0,1,0,0]},"2762":{"type":"single","id":2762,"catId":0,"points":2,"correct":[0,0,1,0]},"2763":{"type":"single","id":2763,"catId":0,"points":2,"correct":[0,0,0,1]}} } });




IMPACT OF CLIMATE VARIABILITY ON CROP YIELD AND WATER SECURITY IN INDIA

TAG: GS 3: ECOLOGY AND ENVIRONMENT

THE CONTEXT: A recent study, published in the International Journal of Water Resources Development, delves into the intricate relationship between climate variability and its effects on crop yield in India.

EXPLANATION:

  • The study offers critical insights into the challenges faced by the agricultural sector, particularly concerning water risks.
  • Here, we will explore the nuanced findings and their implications for informed policy decisions.

Climate Variability and Crop Yield Analysis

  • Seasonal Impact on Crop Yield:
    • The study highlights the significant influence of climate variables like rainfall, evapotranspiration, temperature, and windspeed across different seasons on crop yields.
    • Each crop exhibits varying degrees of sensitivity to these climate elements, impacting both the expected yield and yield variability.
  • Crop-Specific Responses:
    • Crops such as bajra, chickpea, groundnut, rice, and sugarcane demonstrate unique relationships with climate variables.
    • For instance, the optimal levels of rainfall and temperature vary for different crops, influencing their yields positively or negatively.
  • Risk Assessment and Water Elements:
    • Extremes in rainfall, temperature, evapotranspiration, and windspeed are associated with increased yield risks for specific crops.
    • Moreover, the study identifies how low or high values of these climatic factors affect crop yield variability.

Water Management Implications and Policy Considerations

  • Water Availability and Crop Production:
    • Changes in water availability due to climate variations significantly impact crop production.
    • To mitigate adverse effects, the study suggests strategies such as diverting surplus water for storage to meet higher water demands during dry seasons.
  • Addressing Extreme Climate Events:
    • Flooding and droughts caused by extreme rainfall and temperature pose challenges.
    • Effective drainage systems and reallocation of water to crops benefiting from irrigation expansion or tolerant to extremes are recommended for better water management.
  • Adaptive Strategies:
    • The study emphasizes the need for seasonal adaptations and crop-specific measures to sustain long-term water availability.
    • It suggests identifying high-tolerant crops and optimizing irrigation diversification based on climate effects.

Policy Implications and Future Directions

  • Enhancing Water Management Policies:
    • Policy re-evaluation is essential to address the impact of climate change on farm water management.
    • Decision-makers need to consider strategies like predictive techniques, diversification of irrigation, and climate-smart agriculture for sustainable outcomes.
  • Incorporating Scientific Techniques:
    • Implementing modern irrigation techniques and utilizing rainwater effectively can alleviate pressure on groundwater resources.
    • Precision farming, scientific water management, and climate-smart agricultural practices are advocated for resilient crop production.
  • Inclusive Governance and Farmer Participation:
    • Encouraging farmer involvement in water management and aligning policies with region-specific sustainable agricultural practices can yield efficient, equitable, and economically viable outcomes.

Conclusion

  • Understanding the intricate effects of climate variability on crop yield and water security is paramount for reevaluating agricultural policies in India.
  • This study underscores the need for adaptive strategies, scientific water management, and climate-resilient agriculture to mitigate risks associated with climate change.
  • Policy interventions that prioritize sustainable water use and empower local governance can pave the way for resilient agricultural practices and long-term food security in India.

SOURCE: https://www.downtoearth.org.in/blog/agriculture/understanding-climate-effect-on-crop-yield-and-associated-risks-to-water-security-in-india-is-crucial-93393




SPECIAL PROVISIONS FOR STATES IN INDIA BEYOND JAMMU AND KASHMIR

TAG: GS 2: POLITY AND GOVERNANCE

THE CONTEXT: While India’s Constitution tilts towards the Centre on certain areas, not all states are equal either. Right after Article 370, the Constitution creates special provisions for at least nine states, from Article 371A-I.

EXPLANATION:

Quasi-Federal Structure of Indian Governance

  • India’s constitutional framework reflects a quasi-federal structure, balancing the Centre’s authority with varying degrees of autonomy granted to states.
  • The Seventh Schedule of the Constitution delineates Union, State, and Concurrent lists, outlining legislative powers shared between the Centre and states.
  • However, certain provisions tilt authority towards the Centre in specific domains.

Understanding Special Status for States

  • India’s diverse landscape necessitates differentiated approaches in governance, leading to special provisions for various states based on fiscal, political, and administrative considerations.
  • These provisions aim to address regional disparities while fostering unity within the federal structure.
  • However, critics argue that such asymmetric federalism could sow seeds of regionalism and impact national integration.

Examples of Special Status

  • Beyond Article 370:
    • While Article 370 is a well-known instance of asymmetric federalism concerning Jammu and Kashmir, there exist special provisions for nine states, ranging from Article 371A-I.
    • These provisions fall under the Constitution’s section titled “Temporary, Transitional and Special Provisions,” intended to operate until crises like secessionist sentiments or conflicts cease.
    • Importantly, they lack explicit expiration dates.
  • Negotiated Autonomy:
    • States like Nagaland and Mizoram negotiated autonomy with the Centre as a political compromise, safeguarding cultural practices, land ownership, and natural resources from parliamentary interference.
    • These special provisions were pivotal in resolving independence movements in these regions.
  • Delhi’s Unique Arrangement:
    • Delhi, not classified as a state in the Constitution’s First Schedule, operates under Article 239AA, granting it legislative powers over state and concurrent list subjects.
    • This unique arrangement exemplifies a special status designed for the administration of the national capital.

Legal Interpretations and Recent Rulings

  • Challenges and Interpretations:
    • The abrogation of Article 370 in Jammu and Kashmir led to legal challenges asserting that it conferred internal sovereignty, which couldn’t be unilaterally revoked.
    • However, a Supreme Court ruling clarified that Article 370 represented asymmetric federalism, distinct from internal sovereignty.

Conclusion

  • The existence of special provisions for select Indian states reflects the nuanced approach to governance, accommodating diverse needs within the federal framework.
  • While these provisions aim to address regional disparities and political compromises, ongoing debates persist regarding their impact on national integration and the duration of their applicability, signalling the complex interplay between federalism and the unity of the Indian nation.

SOURCE: https://indianexpress.com/article/explained/everyday-explainers/why-many-states-in-india-enjoy-special-provisions-9068631/




COP28 AND THE CLIMATE DEAL

TAG: GS 3: ECOLOGY AND ENVIRONMENT

THE CONTEXT: The recently concluded COP28 in Dubai aimed to address critical issues in combating climate change, particularly the pressing need to align actions with the 1.5-degree Celsius warming threshold.

EXPLANATION:

  • However, an analysis of the agreement suggests a significant gap between aspirations and practical measures to achieve this ambitious goal.

Inadequacies in Addressing Emissions

  • Reality Check on Emissions:
    • Despite claims of progress, the UN Environment Programme’s Emissions Gap Report revealed a disheartening reality.
    • Global emissions in 2022 surged by at least a billion tonnes compared to 2019, indicating a nearly 2% rise, a troubling trend for climate targets.
  • IPCC Recommendations vs. Current Actions:
    • The IPCC guidelines underscore a crucial requirement: a 43% reduction in global greenhouse gas emissions from 2019 levels by 2030 to sustain the 1.5-degree target.
    • However, current actions, as outlined in Nationally Determined Contributions (NDCs), are projected to lead to a mere 2% reduction by 2030, vastly inadequate to meet the urgent emission cut requirement.

Effectiveness of Climate Actions

  • Insufficiency of Annual Emission Reductions:
    • To achieve the necessary 43% cut by 2030, an unprecedented average annual reduction of 8.7% is mandated.
    • Historical data reveals such extensive cuts have never been accomplished, even during significant disruptions like the COVID-19 pandemic.
  • Lack of Immediate Action Acceleration:
    • Despite the agreement highlighting tripling renewable energy and doubling energy efficiency improvements by 2030, there is an evident absence of concrete steps to expedite immediate climate actions.

Challenges and Weaknesses in the Agreement

  • Insufficient Contribution of Measures:
    • Estimates from the International Energy Agency (IEA) suggest that the proposed measures could potentially save 7 billion tonnes of CO2 equivalent by 2030.
    • However, this pales in comparison to the projected shortfall of 24 billion tonnes, as per the Emissions Gap Report.
  • Methane Emissions and Weak Agreements:
    • The agreement’s focus on methane reductions, though impactful due to methane’s potency, is limited as it accounts for less than a quarter of all greenhouse gas emissions.
    • Furthermore, the COP28 agreement on methane cuts lacks defined reduction targets and relies heavily on voluntary commitments.

Financial Constraints and Lack of Resourcing

  • Limitations in Financial Aid:
    • Many developing countries outlined in their NDCs that they could amplify climate actions if provided with requisite financial and technological support.
    • Unfortunately, COP28 failed to address this crucial aspect, hindering the potential acceleration of climate actions.

Conclusion

  • The COP28 agreement, despite its symbolic acknowledgment of transitioning from fossil fuels and emphasizing renewable energy and efficiency improvements, falls short in several critical aspects.
  • The disparity between ambitious climate targets and practical measures to achieve them persists, with insufficient emission reductions and inadequate financial support hampering global efforts to combat climate change effectively.
  • The urgent need for more concrete, aggressive actions remain unmet, jeopardizing the prospect of attaining the 1.5-degree Celsius pathway.

SOURCE: https://indianexpress.com/article/world/climate-change/given-a-robust-action-plan-to-keep-1-5c-target-within-reach-al-jaber-9067394/




PACE MISSION

TAG: GS 3: SCIENCE AND TECHNOLOGY

THE CONTEXT: PACE is a NASA mission scheduled to launch no earlier than Feb. 6, 2024, on a SpaceX Falcon 9 rocket from Space Launch Complex 40 at Cape Canaveral Space Force Station in Florida.

EXPLANATION:

WHAT IS PACE?

  • PACE is NASA’s Plankton, Aerosol, Cloud, ocean Ecosystem mission, currently in the design phase of mission development.
  • It is scheduled to launch in 2024, extending and improving NASA’s over 20-year record of satellite observations of global ocean biology, aerosols (tiny particles suspended in the atmosphere), and clouds.
  • PACE will advance the assessment of ocean health by measuring the distribution of phytoplankton, tiny plants and algae that sustain the marine food web.
  • It will also continue systematic records of key atmospheric variables associated with air quality and Earth’s climate.

PACE has two fundamental science goals:

  • To extend key systematic ocean colour, aerosol, and cloud data records for Earth system and climate studies.
  • To address new and emerging science questions using its advanced instruments, surpassing the capabilities of previous and current missions.

Observatory

  • Goddard Space Flight Center (GSFC) is responsible for the principal mission elements, including the design and fabrication of the spacecraft, development of scientific instrumentation.
  • The Development Team at Goddard Space Flight Center (GSFC) will guide PACE through each phase as the instruments, spacecraft, and observatory are built, tested, and flown.

Observatory Overview

Mass with fuel Not to exceed 1700 kg (3748 lb)
Dimensions 1.5 m x 1.5 m x 3.2 m (4.9 ft x 4.9 ft x 10.5 ft)
Power 1000 Watts
Communications S-Band – Command & Telemetry
Ka-Band – Science Data

INSTRUMENTS:

  • The primary science instruments planned for PACE are:
    • Ocean Colour Instrument (OCI):
      • Spectrometer used to measure intensity of light over portions of the electromagnetic spectrum: ultraviolet (UV), visible, near infrared, and several shortwave infrared bands.
      • The OCI will enable continuous measurement of light at finer wavelength resolution than previous NASA ocean colour sensors, providing detailed information on our global ocean.
      • The colour of the ocean is determined by the interaction of sunlight with substances or particles present in seawater such as chlorophyll, a green photosynthetic pigment found in phytoplankton and land plants.
    • Multi-angle Polarimeters:
      • Radiometers used to measure how the oscillation of sunlight within a geometric plane – known as its polarization – is changed by passing through clouds, aerosols, and the ocean.
      • Measuring polarization states of UV-to-shortwave light at various angles provides detailed information on the atmosphere and ocean, such as particle size and composition.
    • Combined, these instruments will be a major advance in satellite observing technology, allowing for new opportunities to monitor and respond to changes in our ecosystem, and the ways in which the atmosphere and ocean interact.

SOURCE: https://www.nasa.gov/general/experience-the-launch-of-nasas-pace-mission/#:~:text=PACE%20is%20a%20NASA%20mission,biology%2C%20aerosols%2C%20and%20clouds.




NEW DELHI DECLARATION (GPAI SUMMIT)

TAG: GS 3: SCIENCE AND TECHNOLOGY

THE CONTEXT: The Global Partnership on Artificial Intelligence (GPAI) Summit held in New Delhi has concluded with the adoption of the ‘New Delhi Declaration. It  reflected the commitment of 28 countries and the European Union to principles that align with democratic values and human rights in the development, and use of artificial intelligence. India hosted the annual GPAI Summit for the first time.

EXPLANATION:

  • This declaration reflects a recognition of the importance of ethical considerations and accountability in AI technologies.
  • This indicates a focus on ensuring that AI technologies are developed and deployed in a manner that considers long-term implications and societal well-being.

Commitment to democratic values and human rights

  • The GPAI aims to be an inclusive movement, involving countries from the Global South and ensuring that the benefits of AI are accessible worldwide, aligning with the goal of global collaboration.
  • The participating countries acknowledge and commit to addressing various challenges associated with AI, including concerns about misinformation, lack of transparency, fairness, protection of intellectual property and personal data, and potential threats to human rights and democratic values.

Innovation in Agriculture

  • The declaration specifically recognizes India’s contribution to making agriculture a thematic priority for
  • This reflects a recognition of the potential of AI innovation in supporting sustainable agriculture.

International Cooperation

  • The summit emphasizes the importance of international cooperation to find the best possible solutions for the deployment and governance of AI.
  • There is an expressed desire for GPAI to be more inclusive, encouraging the participation of developing countries as well as inclusion of lower and middle-income countries.

MEANING OF ARTIFICIAL INTELLIGENCE GOVERNANCE

Set of policies, regulations, ethical frameworks, and practices that guide the development, deployment, and use of artificial intelligence (AI) technologies.

Ethical and Responsible AI Development

  • AI governance frameworks aim to establish ethical guidelines for the development of AI technologies.

Risk Mitigation

  • Mitigating risks associated with AI technologies like risks related to data privacy, security, unintended consequences, and potential negative impacts on individuals and society.

Transparency and Accountability

  • Transparent AI models are easier to understand, audit, and interpret.
  • An accountable AI governance framework addresses issues of liability in cases where AI systems cause harm or make incorrect decisions.

Inclusive Decision-Making

  • It involves involving diverse stakeholders, including experts, policymakers, industry representatives, and members of the public, to ensure a broad range of perspectives and avoid concentration of power.

GLOBAL AI FRAMEWORK CHALLENGES

Dynamic Nature of AI Technology

  • AI is a rapidly evolving field, and staying updated with the technological advancements is a constant challenge for regulators and policymakers.
  • Ensuring that regulations remain relevant and adaptable to new developments is essential.

Skills Gap and low public awareness

  • There is a shortage of professionals with the necessary expertise in AI governance, policy-making, and ethical considerations.
  • Building public awareness and engagement regarding AI technologies, their implications, and the potential risks is critical.

GLOBAL PARTNERSHIP ON ARTIFICIAL INTELLIGENCE (GPAI)

About the Initiative

History

  • The partnership was first proposed by Canada and France at the 2018 G7 summit, and officially launched in June 2020.
  • India is a founding member of GPAI, but the group does not include India.

CONCLUSION

  • Members of GPAI agreed on the ‘New Delhi Declaration,’ showing a global commitment to using Artificial Intelligence (AI) responsibly, in line with democratic values and human rights.
  • The summit highlighted the importance of an inclusive approach, welcoming participation from developing nations and ensuring that the benefits of AI are accessible to people worldwide.
  • The declaration aims to address concerns related to AI, such as misinformation, transparency, and the protection of intellectual property and personal data.

SOURCE:https://www.thehindu.com/news/national/ai-summit-adopts-new-delhi-declaration-on-inclusiveness-collaboration/article67635398.ece




GST RATES’ RATIONALISATION BACK ON TABLE

TAG: GS 3 INDIAN ECONOMY

CONTEXT: The government has reconstituted the ministerial group of the GST Council that was tasked with recommending the simplification of the complex tax structure and a rejig of its multiple rates.

EXPLANATION

  • Since the beginning of the introduction of the GST tax regime, there has been a call to rationalize the tax structure.
  • The group of ministers (GoM) on GST rate rationalisation, headed by former Karnataka CM Basavaraj Bommai, had been in suspended mode since its formation in 2021.
  • The convenor’s role for the seven-member GoM has been assigned to Uttar Pradesh Finance Minister Suresh Kumar Khanna.

CURRENT GST STRUCTURE

  • The current GST structure has a total of four tax slabs, including 0%, 5%, 12%, 18%, and 28%.
  • There are different forms of taxes levied under GST:

Central GST (CGST)

This tax is imposed on the movement of goods and services within the state but is appropriated by the central govt.

State GST (SGST)

It is the tax levied by the state government and appropriated in the state where the transaction occurs or where the goods are sold and consumed.

ntegrated GST (IGST)

This tax is imposed on all the goods and services between two or more states or union territories.

Union Territory GST (UTGST)

Imposed on supply of goods and services within the Indian Union Territories.

NEED FOR RATIONALISATION

  • Ease-of-doing-business is needed as the current structure is quite complicated. For example, there are some items whose tax rates depend on their packaging, like specified food products, or selling prices.
  • To reduce the number of litigations arising from classification disputes.
  • GST is an indirect tax, which means that it is regressive in nature. Lower tax rates on some of the basic items would ease the pressure on common man.
  • The frequent changes in rates for different goods and services in the past have created uncertainty for businesses.
  • The 15th Finance Commission had observed that the GST’s revenue neutrality was compromised due to multiple tax rate reductions.

IMPLICATIONS

  • Improved compliance: As too many tax rates lead to tremendous compliance-related problems.
  • Improved tax collection: Higher tax rates doesn’t always imply higher revenue collection, going by the Laffer curve theory. This is proven by the fact that with reduction in tax rates on some items in the past, tax collection has only increased.

CONCLUSION

  • Rate rationalization in the GST regime remains an ongoing challenge.
  • There is a need for careful consideration to strike a balance between revenue generation and easing the compliance burden.

Source:https://www.thehindu.com/business/centre-rejigs-gom-to-simplify-gst-rates/article67635249.ece




COP28: TRANSITIONING AWAY FROM FOSSIL FUELS

TAG: GS 3: ENVIRONMENT

CONTEXT: In the recently concluded COP28 Summit, the global leaders took a small step towards moving away from fossil fuels. The adopted Dubai Consensus resolution calls for “just” and  “orderly” acceleration towards renewables.

EXPLANATION

  • It is for the first time that a consensus has been reached to move away from fossil fuels, a major bone of contention for almost 40 years.
  • Many countries, represented by small-island nations, wanted the agreement to be on ‘complete phase-out’. However, in the final document, it was diluted.

WHAT TRANSPIRED?

  • The attitude of middle-income developing countries, who were very uncertain about the much hyped phased out of fossil fuels, was the main reason behind the ‘softening the text’.
  • The host UAE, one of the biggest oil producers, has faced criticism for close ties with fossil fuel interests from the start, especially after oil executive al-Jaber was appointed to preside over the negotiations.
  • This raised the conflict-of-interest issues.

HOW THE TRANSITION WILL BE ACHIEVED?

  • The renewable energy pledge aims to triple the world’s green energy capacity to 11,000 GW by 2030. This agreement has been signed by 118 countries.
  • Countries are also rooting for carbon capture and storage, a technology that oil producers want to use to continue drilling.
  • Another important factor is the requirement for countries to submit stronger carbon cutting plans by 2025.
  • If China and India, among the top polluters, put a rapid transition to green energy at the heart of these new commitments, that could make a massive difference to the global effort.

IMPLICATIONS FOR THE WORLD

  • According to the IPCC, creating a path to net zero greenhouse gas emissions by 2050 is the best shot at keeping global temperatures from rising beyond 1.5 degrees C by the end of the century.
  • This implies cutting emissions to 43% of 2019 levels by 2030 and 60% by 2035.

CHALLENGES TO PLEDGES

  • Lack of greater short-term emissions cuts is the key weakness being cited by experts.
  • Developed countries, even though they have made tall promises, have fallen short of fulfilling them.
  • For example, a 2009 commitment to mobilise $100 billion a year between 2020 and 2025 has only been partially realized.
  • Most of the funding provided by the developed countries to developing countries for transition have been in the form of loans and not grants.

INDIA-SPECIFIC CHALLENGES

  • Developed nations, in over a century of fossil fuel usage, have already used the available carbon space. This leaves very little space for countries like India, whose per capita use of energy is way behind developed countries’.
  • The poor financial condition of power distribution companies (discoms) remains the most important challenge for further scaling up renewables in India.
  • India has been heavily dependent on importing solar panels.
  • There are issues of rising costs, labor constraints, and supply chain bottlenecks, which are impeding renewable energy projects.

CONCLUSION:

  • The consensus on ridding the world of fossil fuels is remarkable, given that it is for the first time that the conference yielded some concrete and realistic commitments.
  • Given the limited carbon space, industrialised western nations must cede space by taking on more stringent reductions than they have committed to so far.
  • A global net zero by 2050 should not imply a net zero for all countries at that time. Developed countries, with access to funds and advanced technologies, should take the lead.

Source: https://www.thehindu.com/sci-tech/energy-and-environment/at-dubai-cop28-nations-reach-consensus-to-transition-from-fossil-fuels/article67633151.ece




THE LIMITATIONS OF CCS AND CDR

TAG: GS 3: ENVIRONMENT

THE CONTEXT: The draft decisions taken at COP28 at Dubai have referred to the abatement and removal of carbon emissions using carbon capture and storage (CCS) and carbon-dioxide removal (CDR) technologies.

EXPLANATION:

  • Considering the meaning of the word ‘abatement’ has become an important bone of contention, understanding the meaning and limitations of CCS is important – as also those of
  • At COP28, the term “unabated fossil fuels” has come to mean the combustion of these fuels without using CCS technologies to capture their emissions. Draft decision texts point to a need to “phase out” such unabated fossil fuels. On the other hand, removal has been referenced in the context of the need to scale zero- and low-emission technologies and support forest restoration as a means to promote emission removal.

WHAT ARE CCS AND CDR?

  • CCS refers to technologies that can capture carbon dioxide (CO₂) at a source of emissions before it is released into the atmosphere.
  • These sources include the fossil fuel industry (where coal, oil and gas are combusted to generate power) and industrial processes like steel and cement production.
  • CDR takes the forms of both natural means like afforestation or reforestation and technologies like direct air capture, where machines mimic trees by absorbing CO₂ from their surroundings and storing it underground.
  • There are also more complex CDR technologies like enhanced rock weathering, where rocks are broken down chemically and the resulting rock particles can remove CO₂ from the atmosphere. Other technologies like bioenergy with carbon capture and storage (BECCS) capture and store CO₂ from burning biomass, like wood.

HOW WELL DO CCS AND CDR NEED TO WORK?

  • While their technical details are clear, scientists have questions about the scale at which CCS and CDR are expected to succeed.
  • The Sixth Assessment Report (AR6), prepared by the United Nations Intergovernmental Panel on Climate Change (IPCC), deals with climate mitigation. It relies a lot on the use of CDR for its projections related to the world achieving the goal of limiting the world’s average surface temperature increase to 1.5 degrees C with no or limited overshoot.
  • The emission scenarios that the IPCC has assessed have more than a 50% chance of limiting warming to 1.5 degrees C assume the world can sequester 5 billion tonnes of CO₂ by 2040. This is more than India emits currently every year. There is no pathway to 1.5 degrees celsius in AR6 that doesn’t use CDR.
  • If CO₂ emissions continue at current levels, we will have a 50% chance of exceeding 1.5 degrees C compared to pre-industrial levels in seven years .
  • To achieve the decrease in CO₂ emissions by direct mitigation would be nearly impossible at this point and would require a lot of

HOW WELL DOES CDR WORK?

  • CDR methods like afforestation, reforestation, BECCS, and direct air capture are constrained by their need for land.
  • Land is often considered to be ‘viable’ for planting trees and deploying other large-scale CDR As a result, such CDR projects can adversely affect land rights of indigenous communities and biodiversity and compete with other forms of land-use, like agriculture that is crucial for ensuring food security.

WHAT ARE OTHER PITFALLS OF CCS AND CDR?

  • By removing CO₂ from their environment, there are concerns that CCS and CDR create more ‘room’ to emit the greenhouse gas. In some cases, CCS has also been used to inject captured CO₂ is into oil fields to extract more oil.
  • In future emissions scenarios that the IPCC has assessed, the world’s use of coal, oil, and gas in 2050 needs to decline by about 95%, 60%, and 45% respectively from their use in 2019 to keep the planet from warming by less than 1.5 degrees C. But without CCS, the expected reductions are 100%, 60%, and 70% for coal, oil, and gas by 2050.
  • In a recent paper, an international group of researchers wrote that higher use of CCS and CDR make way for emissions pathways with a higher contribution from gas.

CONCLUSION:

There is need to identify viable and scalable CDR methods and need to figure out financial mechanism for CDR at a large scale in the future.

SOURCE: https://www.thehindu.com/sci-tech/energy-and-environment/carbon-capture-sequestration-science-limitations-ipcc-explained/article67627040.ece




INDIA VOTES IN FAVOUR OF IMMEDIATE CEASEFIRE BY ISRAEL

TAG: GS 2: INTERNATIONAL RELATIONS

THE CONTEXT: India recently voted in favour of a United Nations General Assembly (UNGA) resolution that called on Israel for an immediate ceasefire and the protection of civilians in accordance with international law and the release of all hostages.

EXPLANATION:

  • The United Nations General Assembly voted overwhelmingly for ceasefire and for the release of hostages and for the observance of international law.
  • India was among 153 countries that made up a massive 4/5th majority in the Assembly who voted in favour of the resolution. Only 10 countries, including the U.S. and Israel, voted against the resolution, and 23 countries, mainly from Europe abstained.

ISRAEL AND PALESTINE REMARKS:

  • Despite being “unsatisfied” with the UNGA resolution, Israel said it appreciated India’s support and votes in favour of two proposed amendments of both U.S. and Austria that were not adopted by the UNGA.
  • The Palestinian Ambassador thanked India for support and said that all countries must say “enough is enough”.

INDIA’S STAND:

  • Earlier India has not supported the resolution as the resolution did not include any specific reference to the October 7 terror attacks in Israel by
  • In the explanation of vote (EoV), India did not clarify the reason for India’s shift but said that India “welcomes” the fact that the international community had been able to find a “common ground” to address the situation in West Asia.
  • Two-state solution’: India mentioned the October 7 “terrorist attack” without naming Hamas, the “humanitarian crisis and large-scale loss of civilian lives”. There is a need to observe the international humanitarian law “in all circumstances”, and the effort to find a lasting “two-state solution” to the question of Palestine.

THE UNGA RESOLUTION

  • The UNGA resolution “Protection of civilians and upholding legal and humanitarian obligations”, was introduced by Egypt and co-sponsored by 21 countries.
  • It was followed by the U.S.’s decision to veto a similar resolution calling for an immediate ceasefire in the N. Security Council.
  • The U.S. decision was unusual as it came despite an explicit invocation of “Article 99” by U.N. Secretary General that the situation in Gaza “threatens the maintenance of international peace and security,”. Article 99 has not been invoked by UNSG since 1989 and the war in Lebanon.
  • “The gravity and complexity of what the international community faces is underlined by the Secretary General invoking Article 99 of the Charter of the United Nations.

IMPLICATION OF THE RESOLUTION:

  • While the UNGA does not have the power to enforce its resolutions as the UNSC does, it carries the sentiment of the global community against the Israeli bombardment of Gaza that has rendered more than a million people homeless.

RESOLUTION BY USA AND AUSTRIA:

  • At the UNGA the U.S. also proposed an amendment that would insert a specific mention of “heinous terrorist attacks by Hamas that took place in Israel starting 7 October 2023 and the taking of hostages”.
  • Austria proposed an amendment to name Hamas and other groups that are holding Israeli hostages.
  • India voted in favour of them, but both amendments were dropped as they did not receive the votes required.

CONCLUSION:

Both Palestinian and Israeli civilian populations must be protected in accordance with international humanitarian law. The challenge in this extraordinarily difficult time is to strike the right balance.

SOURCE: https://www.thehindu.com/news/international/india-and-152-other-members-vote-overwhelmingly-at-un-general-assembly-to-demand-a-humanitarian-ceasefire-in-gaza/article67632382.ece




SAIGA ANTELOPE’S REMARKABLE RECOVERY

TAG: GS 3: ECOLOGY AND ENVIRONMENT

THE CONTEXT: The Saiga antelope, which faced a perilous decline since the breakup of the Soviet Union, has demonstrated a remarkable recovery, prompting the International Union for Conservation of Nature (IUCN) to alter its status from Critically Endangered to Near Threatened.

EXPLANATION:

  • Despite disease outbreaks and challenges, conservation efforts have contributed significantly to its survival.

Saiga Population and Decline

  • In 2003, merely 6% of the Saiga population remained, marking a drastic decline from an estimated one million in the early 1990s.
  • The antelope species, existing since the last Ice Age, were originally spread across the Eurasian Steppe but now inhabit fragmented populations within Kazakhstan, Mongolia, the Russian Federation, and Uzbekistan.

Conservation Impact and Efforts

  • The positive change in the Saiga’s global Red List status is attributed to effective national and international conservation initiatives, particularly in Kazakhstan.
  • Kazakhstan witnessed a substantial recovery in Saiga populations, increasing from a critical low of 48,000 in 2005 to over 1.9 million, largely due to anti-poaching measures, law enforcement, and the establishment of protected areas.

Government and Civil Society Collaboration

  • The Government of Kazakhstan has taken commendable leadership in species recovery, investing in impactful actions and forming collaborations with civil society.
  • This inclusive network involves government bodies, conservationists, academics, and international experts.
  • International organizations like the Convention on the Conservation of Migratory Species of Wild Animals (CMS) and the Convention on International Trade in Endangered Species of Wild Fauna and Flora (CITES) have played significant roles in facilitating coordinated conservation efforts across Saiga habitats.

Ongoing Challenges and Future Prospects

  • While there has been a notable increase in Saiga populations in Kazakhstan and Mongolia, the species still faces challenges.
  • Challenges include ongoing poaching, illegal trade, disease outbreaks, climate change impacts, human interference, and infrastructure development, all of which threaten the Saiga’s full recovery.

Continued Conservation Efforts

  • Efforts to sustain recovery include maintaining population monitoring, protecting key habitats, collaborating with local communities for awareness and conservation actions and addressing persistent threats to Saiga populations.
  • While progress has been significant, the Saiga’s full recovery will only be achieved by restoring its ecological role across its entire range and addressing the persisting threats to its existence.

Conclusion

  • The Saiga antelope’s resurgence from the brink of extinction is a testament to the success of coordinated conservation efforts, particularly in Kazakhstan, and international collaboration.
  • However, sustained efforts are crucial to ensuring the Saiga’s continued recovery and securing its place in the vast Eurasian Steppe ecosystem.

SOURCE: https://www.downtoearth.org.in/news/wildlife-biodiversity/miracle-on-the-steppe-the-saiga-has-beaten-extinction-for-now-finds-new-iucn-red-list-93335